You are on page 1of 67

Cambridge International

AS & A Level Mathematics

Probability & Statistics 1


STUDENT’S BOOK: Worked solutions

Louise Ackroyd, Jonny Griffiths, Yimeng Gu


Series Editor: Dr Adam Boddison

Pure Mathematics 1 International Students Book Title page.indd 1 14/11/17 10:46 pm


57736_Pi_viii.indd 1
WS TITLE PAGE_Probability & Statistics 1.indd 1 6/18/18 10:46
31/07/18 3:21 PM
AM
1
WORKED SOLUTIONS

Worked Solutions
1 Representation of data
Please note: Full worked solutions are provided as an aid to learning, and represent one approach to answering
the question. In some cases, alternative methods are shown for contrast.
All sample answers have been written by the authors. Cambridge Assessment International Education bears no
responsibility for the example answers to questions taken from its past question papers, which are contained in this
publication.
Non-exact numerical answers should be given correct to 3 significant figures, or 1 decimal place for angles in
degrees, unless a different level of accuracy is specified in the question.
Use graph paper when drawing graphs; graphical solutions given here are not to exact scale but give an indication
of the answer.
Prerequisite knowledge Exercise 1.1A

1 a The data is quantitative, as it is numerical, and 1 a Daily rainfall in Penang is continuous, as you
discrete, as there are distinct individual values. measure it.
(The original data could have been continuous, b Monthly texts you send on your mobile is
and then rounded to the nearest mark.) discrete, as you can count them.
b Median = 35, mode = 41 and range = 35 c The number of burgers sold in a fast food
c restaurant is discrete, as you can count
Grouped scores Frequency
them.
10–19 6
d The duration of a marathon is continuous,
20–29 13 as you measure it.
30–39 13 e The age of a teacher in your school is
40–49 18 continuous, as time is measured. But if
someone is asked their age, they reply with
A histogram with equal class widths is a
a whole number, and that version of ‘age’ is
sensible way to display the data.
discrete.
2 a Gender is qualitative.
20 b Height is quantitative, as it has a numerical
Frequency

15 meaning and is continuous.


10 c An IGCSE grade in maths as a letter is
5 qualitative.
d Examination scores in maths are quantitative
0
9.5 19.5 29.5 39.5 49.5 and discrete.
Grouped scores
e Waist size is quantitative and continuous,
unless it is given to the nearest cm/inch, in
The results show a modal group of 40–49.
which case it is discrete.
2 Edam: 90 × 120 = 30 , Stilton: 102 × 120 = 34, f ‘Car owner or not’ is qualitative.
360 360
168 × 120 = 56 g Weekly self-study time is quantitative and
Cheddar: 360
continuous unless measured to the nearest
3 hour, in which case it is discrete.
Score 1 2 3 4 5 6
3 a 9
Frequency 3 4 2 3 3 5
b 9
Mode = most common value = 6 c 13.05
Mean = sum of all the values divided by 20 = 74 = 3.7
20 d The mean takes into account all the visits
Median = middle value when results are ordered = 4 made, so it is the best measure to use.
Range = top result − bottom result = 6 − 1 = 5 1
©HarperCollinsPublishers 2018 Cambridge International AS & A Level Mathematics: Probability & Statistics 1 9780008257767

57767_P001_013.indd 1 04/07/18 9:35 PM


1 Representation of data

4 a Farah’s first week sums to 22.4 mm. 10 a Median = 39th value = 3 years
22.4 + 84.6 = 107, and 107 = 3.82 mm Mode = 2 years
28
Mean = 276 ÷ 77 = 3.58 years
b No effect, since 9.4 + 0.5 = 4.9 + 5.0.
b The quartiles LQ, median and UQ divide the
5 Number of Number of ordered data into four equal sets.
times stopped journeys fx LQ = 19.5th value = 2, UQ = 58.5th value = 5,
1 3 3 IQR = 3
2 5 10 c
3 11 33
4 21 84 1 2 3 4 5 6 7 8 9 10
5 22 110
6 17 102 Exercise 1.2A
7 14 98 1 Range = 41, Q1 = 20, Q3 = 44.5, so IQR = 24.5
8 7 56
Total 100 496
2 ∑ x = 1705,∑ x 2 = 537 615 so
( ) = 22 920 (4 s.f.), s.d.(x) = 151 (3 s.f.)
2
537 615 1705
Median = 100 + 1 = 50.5th value, so 5 var(x) = −
15 15
( )
2
2 537 615 1705
var(x) = − = 22 920 (4 s.f.), s.d.(x) = 151 (3 s.f.)
Mode = 5 15 15
Mean = 496 = 4.96 3 For example, −a, −a, −1, −1, −1, 0, 1, 1, 1, a, a
100 where a = 42.5 = 6.52 (3 s.f.)
 he best average to represent the data is the mean,
T
IQR = 2, s.d. = 4
as it uses all the data, which contains no outliers.
a Mean = ∑ f × mid-point and so
4 Range = 20 – 5 = 15
6
∑f IQR = 14 – 8 = 6
6187.5 ÷ 320 = 19.335… = 19.3 ∑ x = 191,∑ x 2 = 2275
( )
b Since all the mid-points increase by a factor of 2275 − 191
2
1.1, the mean increases by a factor of 1.1. σ=
s.d.(x) = 3.71
18 18
c Since all the mid-points increase by a, the mean
There is an argument for the IQR, since this is
increases by a.
unaffected by the high value of 20. On the other
d a = 0.1 × 19.3 = 1.93 hand, the standard deviation does utilise all the data.
7
Height, h (cm) Frequency fx 5 Mean = ∑ xi
= 3985.1 = 332.091... = 332.1 cm 3
100 < h  120 5 550 12
12

Variance = ∑ i − x = 1 323 540.07 − 3985.1 ( )


2 2
x 2
120 < h  140 4 520 n 12 12
140 < h  160 12 1800 = 10.1307… and so standard deviation = 3.18 cm3
160 < h  180 13 2210 6 Method 1
180 < h  200 8 1520 2 2
 1 − 31  +  4 − 31  +  5 − 31  +  6 − 31 
2 2
 5   5   5   5 
Total 42 6600
2
Estimate of the mean is correct: 6600 = 157.14 +  15 − 31  = 110.8
42  5
8 24 × 84.1 + 4 × 240.3 + 51 × 99.2 = 101.8 kg f × mid-point
79 σ = 110.8 = 4.71 (3 s.f.)
s.d.(x)
9 i Answers to a, c and d are qualitative; answers to 5
b, e and f are quantitative. Method 2
ii Answer to b is discrete; answers to e and f are
( )
2
303 − 31
continuous. ∑ x = 31, ∑ x 2 = 303, s.d.(x) = 5 5
= 4.71 (3 s.f.)
iii Check students' answers

2
©HarperCollinsPublishers 2018 Cambridge International AS & A Level Mathematics: Probability & Statistics 1 9780008257767

57767_P001_013.indd 2 04/07/18 9:35 PM


Variance of combined samples 1
WORKED SOLUTIONS
∑x + ∑ y
( )
2 2 2
= − a+b
2n 2

∑b = 64.4, ∑ g = 68.6 ⇒ ∑b = 1030.4, ∑ g = 891.8 = n(b + a ) + n(a + b ) − ( a + b )


2 2 2
7 2n 2
16 13
2 2 2 2
⇒ ∑ b, g = 1922.2 ⇒ b, g = 66.3 (3sf
. .) = b + a + a + b − a + ab + b
2
2 4
10.2 =
∑ b 2 − 64.42 ⇒ ∑ b 2 = 66 520.96, 2 2 2
= b+ a + a+ b −a
2 2 2 2
− 2ab − b 2
16 4

9.1 =
∑ g 2 − 68.62 ⇒ ∑ g 2 = 61 295.78 = 2b + 2a + a 2
− 2ab + b2
13 4 4
⇒ ∑ b 2, g 2 = 127 816.74
2
= a + b + (a − b)
2 4
( ) = 14.1 (3 s.f.)
2
127 816.74 1922.2
⇒ var(b, g ) = − For the combined samples,
29 29
variance – mean =
8 a This is FALSE. The variance could be 0.64, in
which case the standard deviation would be 0.8. a + b + (a − b)2 − a + b = (a − b)2  0
2 4 2 4
b This is TRUE. The range tells you the span of so variance  mean.

all the data, while the IQR tells you where the
central 50% of the data lies. 10 a The range has the advantage that it is easy to
c This is FALSE. Given a set of data, we can calculate, but the disadvantage that it is affected
subtract the same number from every member significantly by extreme values. The company
of the set to create a new set with a smaller may have a few very highly paid people that
mean but with an unchanged variance. could distort the picture.
d This is FALSE. The data set 1, 2, 3, 4, 5 has b The interquartile range is more sensible than
IQR = 3 and variance = 2. the range as a measure of spread since it gives
you the range for the central 50% of the data,
9 L
 et x be the number of sweets in a bag sampled on
so is unaffected by extreme values. However,
the first day, and y the number of sweets in a bag
sampled on the second day. in concentrating on the central 50%, it has the
disadvantage of ignoring 50% of the data.
a  ∑ x = 300,∑ y = 360,∑ x 2 = 3360,∑ y 2 = 4620 c The variance includes all the data in its
calculation, which is an advantage, but extreme
So mean(x, y) = 11, var(x, y) = 12
values can still have a distorting impact.
b In general ∑ x = na,∑ y = nb,∑ x 2 = n(b + a 2),
d You can say the same for the standard deviation
as the variance, but one advantage of the s.d.
∑ y 2 = n(a + b 2) over the variance is that its units are the same as
2
So mean (x, y) = a + b , var(x, y) = a + b + (a − b) for the data.
2 2 4
(a − b ) 2 11 a She needs to score 74.
so var(x, y) – mean(x, y) = 0
4 b 
24, 34, 37, 39, 42, 54, a
Let x represent data values from the first
sample, and y data values from the second 20
sample. She needs to score anything more than 54.
Then ∑ x = na, ∑ y = nb c   7 
2
9302 + a 2 −  230 + a  = 112 ⇒ a = 57
7
∑ x 2 − a 2 = b ⇒ ∑ x 2 = n(b + a 2) She needs to score 57.
n
∑ y 2 − b 2 = a ⇒ ∑ x 2 = n(a + b 2)
( ) ⇒ a = 71
2
9302 + a 2 −  230 + a  = 10 2 2
d   7 
n 7

Combined mean = ∑
x + ∑ y na + nb a + b She needs to score 71.
= =
2n 2n 2 Exercise 1.3A

Variance of combined samples
1 Various ideas such as height, weight, number of
∑x2 + ∑ y2 −
( )
2
= a+b
texts sent, number of music tracks owned, etc.
2n 2

= n(b + a ) + n(a + b ) − ( a + b )
2 2 2 Discrete variables are counted, while
2n 2 continuous data is measured.
2 2 2 2
= b + a + a + b − a + 2ab + b
2 4 3
2 2 2 2
= 2b + 2a + 2a + 2018
©HarperCollinsPublishers 2b −Cambridge
a − 2ab −International
b AS & A Level Mathematics: Probability & Statistics 1 9780008257767
4
2 2
= 2b + 2a + a − 2ab + b
4 4
57767_P001_013.indd 3 04/07/18 9:35 PM
a + b (a − b)2
1 REPRESENTATION OF DATA

2 For a histogram, the frequency a bar represents is b Median is similar but Molly’s is slightly higher,
proportional to its area. suggesting that she scores more. Larger range for
Since 47 goes with an area of 8 cm2, one bean is Molly, suggesting that Jenson is more consistent
represented by 8 cm2. overall. On the other hand, Jenson has a slightly
47
Since data is measured to the nearest cm, the larger IQR, suggesting that Molly’s middle 50% of
true class intervals are: scores are more consistent.

2.5–8.5, 8.5–13.5, 13.5–25.5 5 a Mode = 60


These have widths of 6, 5, and 12 respectively. b Q0: 36, Q1: 46, Q2: 56, Q3: 64, Q4: 73
The width of 6 goes with the given width of (IQR = 18)
2 cm, so the other classes have widths 5 cm c
3
and 4 cm. The 22 beans in the second class are
represented by an area of 8 × 22 cm2 = 3.75,
47
and dividing by 5 gives a height of 2.25 cm. 30 40 50 60 70 80
3
a Width = cm5 Test mark
3
b Height = 2.25 cm
6 Price of Class Frequency
Frequency
3 a We can treat the data as being continuous printer ($) width density
here. Grouping the data means a histogram 50 < x  70 24 20 1.20
is appropriate.
70 < x  100 31 30 1.03
b Because it includes both start and end values.
100 < x  130 42 30 1.40
c
True distance Number of Frequency 130 < x  160 16 30 0.53
(km) people density
160 < x  200 18 40 0.45
67 = 6.70
39.5–49.5 67 10 200 < x  300 4 100 0.04

124 = 12.40
49.5–59.5 124 10 1.4
Frequency density

1.2
4023 = 804.6
59.5–64.5 4023 5 1.0

0.8
2981 = 596.2
64.5–69.5 2981 5 0.6

89 = 5.93 0.4
69.5–84.5 89 15 0.2

75 = 1.15
84.5–149.5 75 65 50 100 150 200 250 300
Printer cost ($)
4 a Jenson Q0: 3, Q1: 12, Q2: 31, Q3: 55, Q4: 66 The histogram has a hump to the left and a tail to
Molly Q0: 12, Q1: 17, Q2: 34, Q3: 57, Q4: 98 the right. Most of the printers cost less than $150,
but the small number of more expensive printers
means the mean price will be above the median
price.
Jenson
7 a i True
Molly ii True
iii False. The IQR for B is smaller than that for A.
0 10 20 30 40 50 60 70 80 90 100 iv False. Q3 − Q2 is larger for B.
Number of runs

4
©HarperCollinsPublishers 2018 Cambridge International AS & A Level Mathematics: Probability & Statistics 1 9780008257767

57767_P001_013.indd 4 04/07/18 9:35 PM


1
WORKED SOLUTIONS

b There is a larger range for Class B than for Exercise 1.4A


Class A. The IQR for B is smaller than that for
A, however, so the middle 50% in class B is 1
Number in Number of Cumulative
more tightly grouped. The medians for the family families frequency
two student groups are close, suggesting that 1 15 15
attainment is roughly equal for the two groups.
2 20 35
8 a
3 22 57
Mass of Class Frequency
Frequency 4 23 80
raisin (g) width density
0 < x  0.5 4 0.5 8 5 11 91

0.5 < x  1.0 8 0.5 16 6 4 95

1.0 < x  2.0 14 1 14


Median = 48th value, and is approximately 2.6
2.0 < x  3.0 6 1 6 from the cumulative frequency diagram.
3.0 < x  3.5 4 0.5 8
×
90 ×
20
80 ×

Cumulative frequency
Frequency density

70
15
60
×
10 50

40
×
5 30

20
×
10 Median = 2.6
0.5 1 1.5 2 2.5 3 3.5
Mass of raisin (g) 0
1 2 3 4 5 6
Number in family
b The 0.5–1.0 class
9 Before After
Upper Cumulative
65 0 2 Kick distance, l (m)
88776420 1 5589 bound frequency
955333310 2 123566899 5  l < 10 10 5
55420 3 0004467789
630 4 089 10  l < 20 20 58
5 0 Key: 4 2 3 = 24 marks before,
23 marks after 20  l < 30 30 87
30  l < 50 50 102
The median mark for the group has risen from
50  l < 70 70 113
23 to 30; the lesson seems to have had an effect
(although there could be other reasons for the 70  l < 100 100 120
increase).
10 You have Q1 − 1.5 × (Q3 − Q1) = 3,
Q3 + 1.5 × (Q3−Q1) = 83

Rearranging gives 5Q1 − 3Q3 = 6, 5Q3 − 3Q1 = 166


Solving these simultaneously gives Q1 = 33, Q3 = 53

5
©HarperCollinsPublishers 2018 Cambridge International AS & A Level Mathematics: Probability & Statistics 1 9780008257767

57767_P001_013.indd 5 04/07/18 9:35 PM


1 REPRESENTATION OF DATA

a b a = 15, b = 85, central 70% of results lie in


(31,72).
120
c Mean = ∑ fx = 50 855 = 50.9
110 ∑ f 1000
100 The mean and median are similar, so the
distribution is fairly symmetrical.
Cumulative frequency

90

80 4 a Mode = 60
70 b Q1: 46, Q2: 56, Q3: 64
60
(IQR = 18)
50
c
40 50

30 45

20 40

Cumulative frequency
10 P40 = 18 35

30
0
10 20 30 40 50 60 70 80 90 100 110 120
25
Kick distance, l (m)
20

b 40 × 120 = 48th value ≈ 18 15


100
10

c Kick Mid- Frequency fx 5


distance, point 0
l (m) 30 35 40 45 50 55 60 65 70 75 80
Test mark
5  l < 10 7.5 5 37.5
10  l < 20 15 53 795 d P90 − P10 = 69 – 41 = 28 marks. This measure
20  l < 30 25 29 725 is useful because it omits extreme values.

30  l < 50 40 15 600 5 Number of


50  l < 70 60 11 660 0 1 2 3 4 5 6 7 8
goals
70  l < 100 85 7 595
Frequency 7 9 15 9 6 7 4 4 2
Total 120 3412.5
Cumulative
Mean ≈ 3412.5 ≈ 28.4 7 16 31 40 46 53 57 61 63
120 frequency

3 a Median ≈ 48
60
Cumulative frequency

50

1000 40
Cumulative frequency

850 30

20

500 10

1 2 3 4 5 6 7 8
Number of goals
150
The median value is slightly more than 2 goals
0
per game.
50 100
Number of aphids

6
©HarperCollinsPublishers 2018 Cambridge International AS & A Level Mathematics: Probability & Statistics 1 9780008257767

57767_P001_013.indd 6 04/07/18 9:35 PM


1
WORKED SOLUTIONS

6 a Upper class values Cumulative


90
frequency
80

Cumulative frequency
18.0 15 70
18.5 42 60

19.0 60 50
40
20.0 72
30
25.0 87
20
30.0 91 10
b
25 30 35

100 Time (seconds)


Cumulative frequency

90
80 15 × 82 = 12.3,
You need P15 and P85, and
70 100
60 85 × 82 = 69.7
50
100
40 P85 – P15 is approximately 29.7 – 27.6 =
30 2.1 seconds.
20 Median = 18.6cm
9 Only a and f could be parts of a genuine
10
cumulative frequency diagram.
18 20 22 24 26 28 30
A cumulative frequency diagram can never be
Length (cm) decreasing, which rules out b, c and d.
c Median ≈ 18.6 cm A frequency can never be negative, which rules
out e.
d Mean =
∑f × mid-point 1773.45
= = 19.5 (3 s.f.) 10 The maximum value for y is 600, and the minimum
∑f 91 value is zero. The curve is never decreasing and
Thus the mean is greater than the median, starts with a y-value of zero, so it can be a complete
since the bulk of the results are to the left of the cumulative frequency curve.
distribution, with a long tail to the right. The median is the value of x at which y = 320:
7 a 63 minutes 300 − 80 = 5.5
40
b 140 minutes 25 × 600 = 150, 75 × 600 = 450
100 100 , so
c Q3 − Q1 = 80 − 46 = 34 minutes
150 − 80 = 450 = 22.25.
d Approximately 42 minutes Q1 = 1.75, Q3 =
40 20
e Cannot tell using just the box plot. Thus the interquartile range is 20.5.
1.6 hours is 1 h 36 minutes. Using the
cumulative frequency curve, we can draw up Exercise 1.5A
from 96 minutes to the curve, and then read
off approximately 175 on the vertical axis. 1 ∑ x = 56, ∑ x 2 = 560 , mean = 8,
a 

This suggests that about 25 sessions out of standard deviation = 4


the 200 last longer than 96 minutes. b ∑ x = 350,∑ x 2 = 25 500, mean = 70,
f Box plot is more compact, but only shows standard deviation = 14.1 (3 s.f.)
five values (the quartiles), so the cumulative
frequency curve shows more information.
c ∑ x = 91, ∑ x 2 = 1275 , mean = 13,
standard deviation = 3.63
8 Time (seconds) 26 27 28 29 30 31 32 33+ 2 ∑ x = 50, ∑ x 2 = 298 , mean = 5,
a 
Frequency 1 3 15 35 23 4 1 0 standard deviation = 2.19
Cumulative
1 4 19 54 77 81 82 82 b ∑ x = 115.8, ∑ x 2 = 1981.58 , mean = 16.5,
frequency
standard deviation = 3.07

7
©HarperCollinsPublishers 2018 Cambridge International AS & A Level Mathematics: Probability & Statistics 1 9780008257767

57767_P001_013.indd 7 04/07/18 9:35 PM


1 Representation of data

3 ∑ fx = 194,∑ fx 2 = 1216 ,
a  8
2641 175 042
a Mean x = 40 = 66.025 = 66.0 (3 s.f.),var(x) = 40

mean = ∑ fx = 6.06 175 042


∑f x = 2641 = 66.025 = 66.0 (3 s.f.),var(x) = − 66.0252 = 16.749... ⇒ s.d.(x) = 4.09 (3 s.f.)
40 40
2
s.d.(x) = ∑ fx 2 −  ∑ fx  = 1.12
− 66.0252 = 16.749... ⇒ s.d.(x) = 4.09 (3 s.f.)
∑ f  ∑ f  b The smallest possible variance is 0, which
would mean
b Both median and mode = 6.
All three averages are similar, which is due to ∑ x 2 − 66.0252 = 0 ⇒ ∑ x 2 = 174 372.025
40
the symmetrical nature of the distribution.
However, the median or mode is a more ∑ 2 must be a whole number, so the smallest it
x
useful value because it is an actual shoe size. can be is 174 373.
4 ∑ x = 27, ∑ x = 245, n = 13 9 a Mean

∑ x = 27 = 2.08 ∑ fm = 770 + 1750 + 2092.5 + 3335 + 3487.5 + 2040 + 1260


Mean = 200 200
n 13
Standard deviation = = 73.675 = 73.7 (3 s.f.)

∑x2 −  ∑x 
2
245 −  27 
2 ∑ fm 2 − 73.6752 = 1110 737.5 − 73.6752 = 125.682...
n  n  = 13  13 
= 3.81 200 200
 
⇒ standard deviation
⇒ σ = 11.2 (3 s.f.)
Mean = ∑ = 102 = 6.8
x b Since the apple has the mean mass, the mean of
5
n 15 the new batch of apples will be unchanged.
2
1181  102  Since there is now less variation in the masses of
Variance = 15 −  15  = 32.5 (3 s.f.) the apples, the standard deviation of the masses
2 of the new batch will be slightly reduced.
Standard deviation = 1181 −  102  = 5.70 (3 s.f.)
15  15  10 ∑(v − 127.5) ≈ 19 × 0 + 24 × 10 + 35 × 20 + 41 × 30
6 − = x − 10, s.d.(w) = s.d.(x), so
a If w = x – 10, then w + 27 × 40 + 9 × 50 = 3700

x = 10 + 12 = 10.6, s.d.(x) = 144 − 0.6 2 = 2.62 ∑(v − 127.5)2 ≈ 19 × 02 + 24 × 102 + 35 × 202 + 41 × 302
20 20
∑(x − 8) = 10.6 − 8 ⇒ ∑(x − 8) = 52, + 27 × 40 2 + 9 × 50 2 = 119 000
b
20 3700 + 127.5 = 151 (3 s.f.)
Mean ≈
155
6.84 =
∑(x − 8)2 − 2.62 ⇒ ∑(x − 8)2 = 272 (3 s.f.) 119 000  3700 
2
Variance ≈ − = 198 (3 s.f.)
20 155  155 
c y = 48 + a = 10.6 ⇒ a = 9 11 If y = x – 25, then y = x − 25, s.d.(y) = s.d.(x).
30

( )
2
var(y) = 314 − 48 = 7.91 (3 s.f.) (So in fact we
 y = 124 = 6.2, s.d.(y) = 11.8. This implies
30 30 20
do not need to know the value of a here). x = 31.2, s.d.(x) = 11.8.

7 a t = 20, s.d.(t ) = 4.60, c = 20, s.d.(c) = 1.41 12 If y = x – 70, then y = x − 70, s.d.(y) = s.d.(x).

b The country route, as there is a smaller y = −315 = − 6.3, s.d.(y) = 3.41. This implies
50
deviation so it is easier to predict how long it
will take, and is no slower on this evidence. x = 63.7, s.d.(x) = 3.41 (3 s. f.)
2
2106 + p 2  100 + p 
c − = 20, which becomes
6  6 
2
5p – 200p + 1916 = 0
This solves to give p = 16 or 24 (to the nearest
integer).

8
©HarperCollinsPublishers 2018 Cambridge International AS & A Level Mathematics: Probability & Statistics 1 9780008257767

57767_P001_013.indd 8 04/07/18 9:35 PM


2

Variance =
∑ fx 2 −  ∑ fx 
∑ f  ∑ f  1
WORKED
(
779 705 SOLUTIONS
)
2
6 094 1 263 541
= −
1069 1069
= 4 304 297.79
Exam-style questions
s.d. = 4304297.79 = 2074.68
1 a ≈ $2075 (to the nearest dollar)
14 5 5 6 6 8 8
15 0 1 1 2 3 5 6 6 a The mid-points are 3 and 25.5 respectively.
16 0 1 1 2 4 b 7 students are represented by an area of 10  cm2, so
17 0 0 1 student is represented by an area of 10 cm2. The
7
Key: 14 5 = 1.45 m true class width of the 6–10 class is 5 hours, and
the true class width of the 31–49 class is 19, so the
b
width of the 31–49 class on the histogram will be
19
= 7.6 cm. The height of the 31–49 class will be
2.5
10
7
1.4 1.5 1.6 1.7 19 = 0.188 cm.
Height (m) 2.5

c Estimate of the mean = ∑ fx = 602 = 15.05 hours


2 a Median = Q2 = 187, Q1 = 121, Q3 = 260 ∑ f 40
b IQR = 139, Q1 − 1.5 × IQR = −87.5, 7 a
t 5–10 10–14 14–18 18–25 25–40
Q3 + 1.5 × IQR = 468.5
All the 11 values are inside the range (−87.5, Frequency 10 15 22 21 18
468.5), so there are no outliers in the data. Frequency
2 3.75 5.5 3 1.2
3 a Boys density
9 13 4 Girls
322 14 23789
6322 15 256 100
952 16 667 90
63 17 239
Cumulative frequency

80
44 18 Key: 3 14 2 = boys 143,
girls 142 70

60
b 13 4 Girls 50
14 2 3 7 8 9
40
15 2 5 6
16 6 6 7 30
17 2 3 9 20
18
10
IQR for the girls = 167 − 147 = 20
0 5 10 15 20 25 30 35 40 45
c ∑ x = 2392 for current boys. For boys with t (minutes)
new boy added ∑ x = 160 × 60 = 2560.
Thus new boy’s score is 168. b Approximately 86 − 53 = 33 people

4 100 + 2(7) = 114 cm, and 100 − 2(7) = 86 cm. c Mean = ∑ fx = 1643.5 = 19.1 (3 s.f.)
So if the rainfall in the city is outside the range ∑f 86

(86, 114) the year can be taken as exceptional. d Standard deviation ≈

∑ fx = 1 263 541 ∑ fx 2 − x 2 = 37 074.25  1643.5 


−
2

5 a Mean profit = = $1182


∑f 86  86 
∑f 1069
(to the nearest dollar) = 8.12 minutes (3 s.f.)
2
e Q1 ≈ 13
b Variance = ∑ fx 2 −  ∑ fx 
∑ f  ∑ f  Median = Q2 ≈ 18

( )
6 094 779 705 1 263 541
2 Q3 ≈ 23, P60 = 18 minutes
= −
1069 1069
= 4 304 297.79
s.d. = 4304297.79 = 2074.68
9
≈ $2075 (to the nearest dollar)
©HarperCollinsPublishers 2018 Cambridge International AS & A Level Mathematics: Probability & Statistics 1 9780008257767

57767_P001_013.indd 9 04/07/18 9:35 PM


1 REPRESENTATION OF DATA

8 a The median is the 123rd result, which lies in the 10 a


170–180 class. 100 ×
The upper quartile is the 185th result, which 90 ×
also lies in the 170–180 class.
80

Cumulative frequency
b
70

8 60
×
7 50
Frequency density

6 40

5 30

4 20 Median ≈ 20.8
×
3 10

2 × ×
0
12 14 16 18 20 22 24 26
1
Foot length, l (cm)
130 140 150 160 170 180 190 200 210 220
Length of song (sec)
b ∑ fx = 2077,∑ fx 2 = 43524.5
Mean ≈ 20.8 cm
c Mean ≈ ∑ fx = 42445 = 173 sec (3 s.f.) Standard deviation ≈ 1.96 cm (3 s.f.)
∑f 246
c Mean is best as no obvious outliers, and it
9 a The class widths are 200, 40, 40, 70, 150, 300, so
uses all available data.
the frequency densities are 0.03, 0.5, 0.75, 0.343,
0.08, 0.0267. 11 a Mean = 146 +160 = $164.87
30

( )
The area of the 200–240 class is 14, so the 2
Variance = 1024 − 146 ⇒ standard deviation = $3.23
area per household is 0.7, so the area for 30 30

( ) ⇒ standard deviation = $3.23


the 280–350 class is 16.8 cm2. 2
1024 − 146
The width of the 280–350 class is 2 × 70 = 30 30
40
3.5 cm, so its height must be 4.8 cm.
b
b ∑ x = 146 + 160 × 30 = 4946,∑ x 2 = 30
  ∑x  
2
×
100
×
∑ x = 146 + 160 × 30 = 4946,∑ x = 30  var(x) +  30   = 815 744
2

90  
Cumulative frequency

80 × Alternative method: multiplying out, 1024 =


70
∑ x 2 − 320∑ x + 30 × 1602 ⇒ ∑ x 2 = 815 744
60
×
50 c Mean = 4946 = 164.9 (4 s.f.)
30
40
Variance = 815 744 − 164.8666 2
30 30
×
20 ⇒ standard deviation = $3.23 (3 s.f.)
10 Median ≈ $270 12 a
×
×
0
100 200 300 400 500 600 700 800 900 1000 30  40  50  60  80 
Mass, m
Monthly shopping bill ($) m< m< m< m< m<
(grams)
40 50 60 80 120
c ∑ fx = 30 660,∑ fx 2 = 10 984900 Frequency 13 37 56 8 6
30660
Mean ≈ = $307 (3 s.f.) Cumulative
100 13 50 106 114 120
Standard deviation ≈ frequency

10 984 900
− 306.6 2 = $126 (3 s.f.)
100
10
©HarperCollinsPublishers 2018 Cambridge International AS & A Level Mathematics: Probability & Statistics 1 9780008257767

57767_P001_013.indd 10 04/07/18 9:35 PM


1
WORKED SOLUTIONS

So here, the standard deviation is more than


120 three-quarters of the IQR.
Note: if the distribution was more
symmetrical and less skewed, the rule would
work better.
Cumulative frequency

90

13 a Mean = 4930 = 290 kg


17
290 + 2(10.5) = 311 kg
60
b New mean = 4930 + 292 = 290.1 kg
18
2.5 × 50
14 a Width = = 12.5 cm
10
30
b Height = 4 × 34 = 9.7 cm
14
c
×
100 ×
30 60 90 120
90
Mass, m (g) Q3 ≈ 38 ×

Cumulative frequency
80
i Median ≈ 51g ii LQ ≈ 44 g, UQ ≈ 57 g 70
b 60
30  40  50  60  80 
Mass, m 50 ×
m< m< m< m< m<
(grams)
40 50 60 80 120 40

Frequency 13 37 56 8 6 30 Q1 ≈ 21
Class width 10 10 10 20 40 20 ×
Frequency 10
× Median ≈ 29
1.3 3.7 5.6 0.4 0.15
density 0
20 40 60 80 100

Time (milliseconds)
6

5 52nd value = median ≈ 29


26th value = Q1 ≈ 21
Frequency density

4 78th value = Q3 ≈ 38
IQR ≈ 17
3 d Median is best due to the extreme values in
the data set.
2
15 a The given bar has an area of 9 cm2, so each plant
is represented by an area of 9 cm2. That means
29
1
the area of the 25  m < 35 bar is 9 × 4 cm2.
29
The width of the 25  m < 35 bar must be double
30 60 90 120 the width of the 5  m < 10 bar, at 3 cm.
Mass, m (g) That means the height of the 25  m < 35
c For this data, IQR ≈13 g. bar is 9 × 4 = 0.414 cm (3 s.f.).
3 × 29
Mean ≈
∑ fm ≈ 53
120
b Mean = ∑ fx = 10.7 kg
∑ fm 2 − 532 = 13.7 (3 s.f.) ∑f
s.d. ≈
120
Standard deviation = ∑ fx 2 − x 2 = 6.84 kg
∑f
11
©HarperCollinsPublishers 2018 Cambridge International AS & A Level Mathematics: Probability & Statistics 1 9780008257767

57767_P001_013.indd 11 04/07/18 9:35 PM


1 Representation of data

16 a Mean = ∑ fx = 18 a Mean =
310 467
125
= 2484 s (41 minutes 24 seconds)
∑f
2
23 × 105 + 33 × 64 + 43 × 42 + 53 × 30 + 64 × 21 + 80 × 10 773 071 329  310 467 
Variance = − = 15 626.11...
272 125  125 
= 37.011... s.d. = 125 s (2 minutes 5 seconds)
= 37 years, no months
b ∑(t − 2500) = −2033,∑(t − 2500)2 = 1 986 329
∑ fx 2 − x 2 =
( )
2
437185 10 067
s.d. = −
∑f 272 272 19 a Australia
853 48
England
0148
= 15.4 (3 s.f.) 98210 49 233679
= 15 years 5 months (nearest month) 976641 50 1147888
885430 51 246699
b The modal class is the one with the 8611 52 034 Key: 0 48 3 =
highest frequency density. This is the class 542 53 6 48.0 seconds Australia,
containing 18–27 year olds.
48.3 seconds England
c
Age 18–27 28–37 38–47 48–57 58–69 70–89 b
Freq 105 64 42 30 21 10 Australia
CF 105 169 211 241 262 272
England

300
48.0 49.0 50.0 51.0 52.0 53.0 54.0
Time (seconds)
250
c Both diagrams show the data well. Box -and-
Cumulative frequency

200 whisker plots show that overall the English


cricketers have slightly quicker times. Although
Australia has a smaller range, the interquartile
150
range of the two teams is similar. The stem-and-
leaf diagram has the advantage that the original
100 data is preserved.
20 For distribution 1, (lowest value, Q1, Q2, Q3,
50 highest value) = (8, 10, 14, 22, 30)
For distribution 2, (lowest value, Q1, Q2, Q3,
0 highest value) = (8, 14, 22, 26, 30)
15 30 45 60 75 90
Age (years)
 edian ≈ 33, LQ ≈ 24, UQ ≈ 46, IQR ≈ 22 (all
M
in years)
d T
 he modal class would be the most meaningful 0 5 10 15 20 25 30
measure to choose here, although it would help The values for distribution 2 are significantly
to know what percentage of all drivers are in higher, as shown by the differing values for the
each class to make a fully-informed choice. median. The measures of spread, however, are the
17 IQR = 6 same here; the variation in each set of data would
∑ x = 55 + a,∑ x 2 = 385 + a 2 ⇒ seem to be the same.

2 2
s.d. = 385 + a −  55 + a  = 6
11  11 

This gives 10a2 – 110a – 3146 = 0, and so a = 24

12
©HarperCollinsPublishers 2018 Cambridge International AS & A Level Mathematics: Probability & Statistics 1 9780008257767

57767_P001_013.indd 12 04/07/18 9:36 PM


1
WORKED SOLUTIONS

Mathematics in life and work

Drug Patients L Q L C (£ CE Continue?


× 1000)
Q
A 15 1 0.9 0.9 10 11.1 no
B 31 2 0.7 1.4 12 8.6 no
C 64 3 0.7 2.1 15 7.1 yes
D 42 4 0.6 2.4 8 3.3 yes
E 34 5 0.4 2.0 17 8.5 no
F 17 6 0.3 1.8 3 1.7 yes

2 ∑ fx = 2447,∑ fx 2 = 33 031
Mean = 2447 = 12.1 (3 s.f.)
203
Standard deviation =
2
33 031  2447 
− = 4.17 (3 s.f.)
203  203 
3 Median = 102nd value = 3 years
Q1 = 51st value = 3 years
Q3 = 153rd value = 5 years
4 CE is the largest for drug A.
5 NICE would wish to discontinue drugs A, B and E,
and continue with drugs C, D and F .

13
©HarperCollinsPublishers 2018 Cambridge International AS & A Level Mathematics: Probability & Statistics 1 9780008257767

57767_P001_013.indd 13 04/07/18 9:36 PM


2 PROBABILITY, PERMUTATIONS AND COMBINATIONS

2 Probability, permutations and combinations


Please note: Full worked solutions are provided as an aid to learning, and represent one approach to answering
the question. In some cases, alternative methods are shown for contrast.
All sample answers have been written by the authors. Cambridge Assessment International Education bears no
responsibility for the example answers to questions taken from its past question papers, which are contained in this
publication.
Non-exact numerical answers should be given correct to 3 significant figures, or 1 decimal place for angles in
degrees, unless a different level of accuracy is specified in the question.
Use graph paper when drawing graphs; graphical solutions given here are not to exact scale but give an indication
of the answer.

Prerequisite knowledge
b P(both yellow) = 16 = 0.16
100
1 a c P(one blue and one yellow) = 0.24 + 0.24 = 0.48
x

B K Exercise 2.1A

1 a 8! = 40 320
59
24 b 8 × 7 × 6 = 336
30 2 5! × 21! = 6.13 × 1021 (5! arrangements for the
vowels, and then a following 21! arrangements for
the remaining letters.)
? 3 a 8! = 40 320
b The number of ways where the two S books
are together = 7! + 7! = 10 080.
b 120 − 59 − 30 − 24 = 7 members
Alternatively: There are 2! ways to arrange
2 a 1 the two S books. There are 6! ways to
3
arrange the remaining 6 books. Finally there
b 4 are 7 possible positions for the S books in
15
relation to the N books, so 7 × 6! × 2! = 10 080
c 11 ways in total.
15
So the number of ways where the two S
3 a P(K and J) = 0.2
books are not together is 40 320 − 10 080 =
b P(K and H) = 0 30 240.
c P(H′) = 1 − 0.35 = 0.65  2  6   2  6 
c     +     = 55
d P(K′ and J′ and H′)  1   3   2  2
= 1 − (0.25 + 0.2 + 0.1 + 0.35) = 0.1
4 The group must consist of two from one nation
e P(K or H) = 0.25 + 0.2 + 0.35 = 0.8
and one from each of the others.
4 a
36 So the total number of possible choices is:
6 Blue
100  5  4  3
10
 2 × 4 × 3 × 4 + 2 ×  2 × 5 × 3 × 4 +  2 × 5 × 4 × 4
6 Blue = 1440
10 4 24
10 Yellow 5 a The number of ways of arranging all 12 books
100
24 is 12! = 479 001 600.
6 Blue
4 100 The number of ways of arranging the eight Plato
10
10 books is 8! = 40 320, while the number of ways of
Yellow
arranging the four Socrates books is 4! = 24.
4 So the probability of having Plato-Socrates is
10 Yellow 16 8!4! while the probability of Socrates-Plato
100
12!
14
©HarperCollinsPublishers 2018 Cambridge International AS & A Level Mathematics: Probability & Statistics 1 9780008257767

57767_P014_025.indd 14 7/5/18 3:39 PM


2
WORKED SOLUTIONS

is 4!8! , and so the probability of having all m(m − 1) n(n − 1) = 9 n(n − 1)(n − 2) m ⇒ (m − )
5 − 2) 1
12! 2 m2(m − 110
) n(n − 1) 6= 9 n(n − 1)(n m ⇒ 5(m − 1)
2 2 10 6
the Plato books andmall − 1)Socrates
(mthe 9 n(n − 1)(n − 2) m =⇒3(5n(m
n(n − 1) =books − 2)
− 1) = 3(n − 2)
2 2 10 6
together is 2 × 8!4! = 2 .
12! = 3(495
n − 2)
So the smallest values are m – 1 = 3, n – 2 = 5, or
b We now need the probability of Socrates- m = 4, n = 7.
Plato-Socrates, where the arrangement is 10 a 3 × 8! = 120 960     b 3 × 3 × 6! = 6480
0–8–4 or 1–8–3 or 2–8–2 or 3–8–1 or 4–8–0. c 5 × 4 × 6! = 14 400
The probabilities here are:
Exercise 2.2A
8! 4! + 4 × 8! × 3! + 4 × 3 × 8! × 2!
12! 12! 12! 1 a 
If there are 200 students in total, 46 must be in
4 × 3 × 2 × 8 ! × 1! 4 ! × 8!
+
12!
+
12! Nis. The probability is therefore 46 = 0.23.
200
= 5 × 8! × 4! = 1 b Probability of ‘Qom’ house = 0.31.
12! 99
Probability of ‘Aba’ house = 0.26.
6 a 6! = 60  robability of not ‘Qom’ house or ‘Aba’
P
2!3!
house is therefore 1 − (0.31 + 0.26) = 0.43.
b 5! × 2 = 20
3!2! 2 a Half of the tickets are even numbers so 1 or 0.5.
2
c 4! = 4
3! b Square ticket numbers are 1, 4, 9, 16, 25, 36, 49, 64,
7 a 
Regarding the Es and Ls and Ss as distinct 81, 100. There are 10 square ticket numbers.
letters, there are 8! arrangements of the letters. Probability of selecting a square ticket
In these, the Ls can be arranged in 2! 10
number = = 0.1.
100
ways, as can the Es and the Ss. Thus the Therefore the probability of not selecting a
total number of distinct arrangements is square ticket number = 1 − 0.1 = 0.9.
8!
= 7! = 5040.
2!2!2! c Ten numbers end in 5, ten start with 5, and
b Count instead the number of arrangements one number does both, so 19 numbers
where the two Ls DO appear together. feature the digit 5. The probability is 0.19.
This is (regarding LL as one letter) 3 20 = 3x + 8
7! 12 = 3x
= 1260.
2!2! x=4
So the number of distinct arrangements 4
Probability of selecting a black ball = = 0.2.
where the two Ls do not appear together is 20

5040 – 1260 = 3780. Therefore, the probability of not selecting a


black ball = 0.8.
8 a K(H)Q(H) appears in 7! arrangements, while
Q(H)K(H) appears in another 7!, so there are 4 a 6! = 3
2 × 7! ways for this to happen altogether ( )
8!
3!
28
= 10 080 arrangements.
b P(green and yellow together)
b K(S) and K(C ) are together in 10 080
7! × 2 = 1 .
arrangements, so they are not together in
8! – 10 080 arrangements = 30 240
=
( )
3! 8 !
3!
4
3
arrangements. P(not together) = .
4
c If K(H) and Q(H) are together and K(S) and 6!
c 8! = 3
K(C ) are together, then there are (regarding 3!
28
K(H)Q(H) as one card and K(S)K(C) as
 10
another) 6! × 2! × 2! = 2880 arrangements.  
5 a  4 = 2
 m  n  m  n  30 261
9 You have     = 0.9      4 
 2   2  1   3
 n n!
Using   = r !(n − r )!
r
15
©HarperCollinsPublishers 2018 Cambridge International AS & A Level Mathematics: Probability & Statistics 1 9780008257767

57767_P014_025.indd 15 7/5/18 3:39 PM


2 Probability, permutations and combinations

 20 Thus P(team of five contains the goalkeeper,


 4  two defenders and two attackers) = 100 = 50 .
b = 325 462 231
 30 1827
 4   24
9 a   × 4 × 3 = 127 512
 4
 10  13  7
 2  × 13 × 7 +  2  × 10 × 7 +  2 × 10 × 13  14  13  10
c  1   2   1  10 920
 30 65
b = = = 0.0856 (3 s.f.)
 4   24 127 512 759
× 4 × 3
 4 
12 285 13
= =
27405 29  10  22
6 Probability of Alan getting a job (up to age 24)  2   2  × 2 20790 15
c = = = 0.163 (3 s.f.)
≈ 1 − 0.132 = 0.868.  24 127512 92
 4  × 4 × 3
The above figure is a crude estimate. Is Alan
better or worse qualified than the average
unemployed Australian? Does he spend more 3
10 a P(H) =
4
or less time than the average seeking work? Are
the job opportunities for Australians the same b P((H or T )) = 1 – P(H or T )
as for those from overseas? Is Alan willing to = 1 – P (spade/diamond/club or ten)
= 1 –  3 + 1  = 3
compromise on levels of pay? There are a large
number of unknowns. It could be that since  4 52  13
Alan is choosing to move to Australia, he may c P((B or (T or H))) = 1 – P(B or (T or H ))
have specific work in mind that increases his
= 1 – P(B or T or H )
chances of finding a job.
7 a = 1 – P(T or H ) (since if B happens, H and T
happen)
Heads Tails
 3 1
Spades SH ST =1– + 
 52 4 
Hearts ♥H ♥T 9
=
Diamonds DH DT 13

Clubs CH CT Exercise 2.3A


All eight outcomes are equally likely.
1 x
b Red suits = hearts and diamonds, so the
probability is 2 = 1 A B
8 4
c
0.3
H, H H, T T, H T, T 0.1
0.2
Spades SHH SHT STH STT
Hearts ♥HH ♥HT ♥TH ♥TT
Diamonds DHH DHT DTH DTT
0.4
Clubs CHH CHT CTH CTT
All 16 outcomes are equally likely. a 0.6
2 1 b 0.1
d =
16 8

8 Total number of choices for the team of five is


11
C 5 = 462 .
Total number of teams with the goalkeeper, two
defenders and two attackers = 5C 2 × 5C 2 = 100 .

16
©HarperCollinsPublishers 2018 Cambridge International AS & A Level Mathematics: Probability & Statistics 1 9780008257767

57767_P014_025.indd 16 7/5/18 3:39 PM


2
WORKED SOLUTIONS

2 x 5 x Pianists

A B 42
violin guitar
0.15
0.35
0.15 36 10 68

0.35

a 0.7
b 0.5 P(piano but not violin and not guitar) = 42 = 7
156 26
c 0.65
6 x
3 x
J L
K S
0.15
x 0.25
z 0.25
y

0.35
w
P( J or L but not both) = 0.4.
x + y = y + z = 3y, x + y + z = 0.75,
so 5y = 0.75, y = 0.15, x = z = 0.3 7 x Gardeners
a 0.15
b 0.45 hoe fork
c 0.55 10 x
24
d 0.25 x 7 x
e 0.3
14
4 x 9
A B spade

3x + 64 = 100, so x = 12.
0.1
0.2
0.4 P(spade) = 14 + 12 + 7 + 12 = 0.45
100
8 The probabilities must add up to 1, so 4x + 4y = 1,
x + y = 0.25.
0.3 P(B) = 0.3 implies 3y + x = 0.3.
Subtracting the two equations,
a 0.4
2y = 0.05, so y = 0.025, x = 0.225
b 0.1
Thus P(C) = 3x + y = 0.7.
c 0.2

17
©HarperCollinsPublishers 2018 Cambridge International AS & A Level Mathematics: Probability & Statistics 1 9780008257767

57767_P014_025.indd 17 7/5/18 3:39 PM


2 Probability, permutations and combinations

9 Say the number of members doing all three Exercise 2.4A


Saystrokes
the number
is a. of members doing all three strokes is a.
1 x
x
A B
crawl butterfly
30–(39–a) 24–a
33–(43–a)
a 1 1
15–a 19–a 5 4
30–(34–a) ?
11
backstroke
20
So 7 = 3a–23 and so a = 10
a 0
x 1
b 5
crawl butterfly
11
c 20
1 14
0
10 2 A and B are mutually exclusive if and only if
5 9
P(A ∪ B) = P(A) + P(B).
6 5 1
Let A be ‘the sum on the dice is 4’, so P(A) = .
12
backstroke Let B be ‘the scores are equal’, so P(B) = 1 .
6
a P (all three strokes) = 10 = 1 P(A ∪ B) = 8 = , which does not equal 1 + 1 = 1 .
2
50 5 36 9 12 6 4
5 Thus A and B are not mutually exclusive.
b P (none of these) = =1
50 10 3 1
B
2
10 x 5 B
9 1 Y
A 2
B
5
B
ar 4 8
ar 3
ar 2 Y
9
3
Y
8
a a 5
8
b 1
2
8
a + ar + ar2 + ar3 = 1 ⇒ a = c 4×3=1
15
9 8 6
Thus P(A) = 1 and P(B) = 1
3 5 d 4×5+5×1=5
9 8 9 2 9
11 a Wears a Does not Total 4 a 2 8
scarf wear a scarf 3 H 21
Wears 5 1 2 R
1 4
a hat 12 4 3 4 T
3 21
7
Does not 1 1 1 2 1 1
2 H 7
wear a hat 12 4 3 7
B
Total 1 1 1
1 1 T
2 2 7
2
1 1 1
7 2 H 14
1
P ( S ∩ H ′ ) 12 1 G
b = =
P ( H ′) 1 4 1
T
1
3 2 14
18
©HarperCollinsPublishers 2018 Cambridge International AS & A Level Mathematics: Probability & Statistics 1 9780008257767

57767_P014_025.indd 18 7/5/18 3:39 PM


2
WORKED SOLUTIONS

8 2 7 1 4 4
b P(red and heads) = a 7 × 13 + 2 × 13 = 13
21
1 b P(at least one blue)=
P(blue and heads) =
7
1 – (P(RR) + P(GG) + P(RG) + P(GR))
1
P(green and heads) = 2 3 1 6 4 36
14 = 1− × − × − =
7 13 2 13 13 91
8 1 1 25
P(heads) = + + = = 0.595
21 7 14 42 c P(both the same) = P(RR) + P(GG) + P(BB)
5 0.4 W 2 3 1 6 3 2 30
= 7 × 13 + 2 × 13 + 14 × 13 = 91
0.45 L
0.6 B
P (different) = 1 – 30 = 61
B 91 91
0.6 0.1 W
0.55 S 8 a x
0.9 B
0.5 W
0.25 L bus train
0.4 0.5 B
FR 45
6 18
0.2 W
0.75 S
0.8 B 75
a 0.6 × 0.45 × 0.4 + 0.4 × 0.25 × 0.5 = 0.158
b 0.6 × 0.45 × 0.6 + 0.6 × 0.55 × 0.9 + 0.4 × 6
P(B ∩ T ) = = 0.0417
0.25 × 0.5 + 0.4 × 0.75 × 0.8 = 0.749 144

c 0.4 × 0.25 × 0.5 + 0.4 × 0.75 × 0.8 = 0.29 P(B) × P(T ) =


51
144( ) ( )
×
24
144
= 0.0590

6 a i 0.65 × 0.4 × 0.25 = 0.065 These are not equal, so B and T are not
ii 0.2 × 0.85 × 0.75 = 0.128 independent.
iii 0.1 × 0.45 × 0.55 + 0.1 × 0.45 × 0.45 + 0.1 × b x
0.55 × 0.55 + 0.45 × 0.55 × 0.9 = 0.298 (3 s.f.)
b i P(both walk) = 0.9 × 0.25
bus train
P(both cycle) = 0.55 × 0.7
P(same method ) = 0.61 45 6+x 18 – x
ii P(different methods) = 0.39

7 2 B
13 75
7
13 G
B
P(B ∩ T ) = P(B) × P(T ),so

( )
4
(6 + x) (51 + x) 24
3 13 R = × ,
144 144 144
14
3 B which solves to give x = 3
7 13
6 9 a A and B are mutually exclusive.
14 13 G
G b x
4 C
13 R A
4 1
3 B 3
14 6
13 2
7 5
R 13 G
4
B
3
13 R
19
©HarperCollinsPublishers 2018 Cambridge International AS & A Level Mathematics: Probability & Statistics 1 9780008257767

57767_P014_025.indd 19 7/5/18 3:39 PM


2 PROBABILITY, PERMUTATIONS AND COMBINATIONS

Probabilities of 1, 2, 3, 4, 5, 6 are kp, p, p, p, p, p x


5
so p(5 + k) = 1, and p = 1 .
5+k E F
P(B)P(C) = P(B∩C)⇒
k+2 3 2
= ⇒k=4 1
5+k5+k k+5 3
10 7
20
10 Since A and B are mutually exclusive, P(A∩B) = 0. 20
If they are also independent, then P(A∩B) = P(A)P(B).
Thus P(A)P(B) = 0, and so either P(A) = 0 or P(B) = 0 2
or both. 5
Exercise 2.5A
P(E ∩ F )
a P(E | F ) =
1 P(spade) = 1 , P(jack) = 1 , P(spade and jack) = 1 P(F )
4 13 52
= 0.15 ÷ 0.5
1
= 0.3
P(jack | spade) = = 1 52
13 1
4 b P(E′ ∩ F ) = P(F ) − P(E ∩ F )
Or, since the events spade and jack are clearly
= 0.5 − 0.15
independent, P(jack | spade) = P(jack).
6 = 0.35
2 P(W) = 14 , P(W ∩ H) =
31 31 c P(E′ ∩ F′) = 1 − P(E ∪ F )
P (H ∩ W ) = 1 − 0.6
P(H | W) =
P (W )
= 0.4
6 14
= ÷
31 31 6 a 5 Red
3 6
=
7 Yellow
5 Red
1 1 1 1
3 P(H, T or T, H) = , P(H, H) = , P(T, T) = 11
2 4 4 6 4
6 Red
1
P(T , T ) 1 6
a P(T, T | T) = P(at least one T ) = =
4
3 3 Red Yellow Yellow
4 12 6 2
11 6
b P(H, T | T) = P(H ,T or T , H )
P(at least one T ) 6 4
1 6 11 6
Yellow Red Red
= 2 =2 12
3 3
4 2
Yellow
4 x 5 6 3
11 Yellow 6 Red
P M
3 Yellow
0.1 6
0.2
0.4
 1 5 5  1 6 4
b P(B) =  × ×  +  × × 
 2 11 6   2 11 6 

+  1 × 6 × 4  +  1 × 5 × 3 
0.3  2 11 6   2 11 6 
2
a P(M ∪ P) = 0.7 =
3
b P(P | M) = P(P ∩ M ) = 0.4 =
2
P(M ) 0.6 3 c P(A ∩ B) = P(R, R, R) + P(Y, Y, R)

c P(M | P) =
P(M P) = 25 + 5
P(P) 132 44
= 0.4 ÷ 0.5 10
= 0.8 =
33
P(M ∩ P ') 0.2 2
d P(M | P′) = = =
P(P ') 0.5 5
20
©HarperCollinsPublishers 2018 Cambridge International AS & A Level Mathematics: Probability & Statistics 1 9780008257767

57767_P014_025.indd 20 7/5/18 3:39 PM


2
WORKED SOLUTIONS

d (R, Y, Y) and (Y, R, Y) are the only disallowed b P(G) = 0.1 × 0.3 + 0.9 × 0.5 = 0.48
combinations.
P(W ∩ G ) 0.03
So P(A ∪ B) = 1 – 1 − 1 c P(W | G ) = = = 0.0625
11 11 P(G ) 0.48

9 9 a 1 b 0
=
11
c P(A ∩ B) = P(B | A)P(A) = 0.2
25 5 10 P( A ∩ B) 0.2 2
e P(R, R, R) + P(Y, Y, Y) = 132 + 44 = 33 d P(B) = P( A | B) = 0.3 = 3
25
P(all red | same colour) = 132 =5 P( A ∩ B ′ ) P( A ∩ B ′ )
e P( A | B ′ ) = =
( ) 10
33
8 P(B ′ ) 1 − P(B)
7 a P( A) − P( A ∩ B)
3 Heads =
1
5 3
= 3(0.5 − 0.2) = 0.9
2
Red Tails
9 2 1 1
5 10 a 4 = P(R | S) + P(S | R)
1 P(S) P(R)
5 = +
7 Blue Heads P(R ∩ S) P(S ∩ R)
9 P(S) + P(R)
=
4 P(R ∩ S)
5 Tails 0.8
=
P(R ∩ S)
2 2
(
7 4
b P(tails) = 9 × 5 + 9 × 5 )( ) Thus P(R ∩ S) = 0.2.
5 1 8 2
32 b 4= + = ⇒ P(S | R) =
= 3P(S | R) P(S | R) 3P(S | R) 3
45
P(R ∩ S) 0.2
c P(heads) = 13 P(R ) =
P(S | R)
=
2
= 0.3, so P(S) = 0.5
45
3
P(blue ∩ heads)
P(blue | heads) =
P(heads)
7 13 Exam-style questions
= ÷
45 45
1 a 1 − 56 = 0.65
7 160
=
13
b 24 + 32 = 0.35
d Probability of same colour = P(red and red) + 160
P(blue and blue) c
( 3060 ) = 30 ÷ 48 = 0.625
( )( ) ( 160 )
48
2 2 7 7
= × + ×
9 9 9 9
 3   4   5  3   4   5 
53 2 a   ×  ×  +  ×  × 
=  2  2   3  2  3   2
81
 3  4   5 
8 a G +   ×   ×   = 360
0.3  3  2   2

W b 10! × 3! = 2177 2800


0.1
0.7 3 a P(Uche does not) = 1 − 0.62 = 0.38
G'
P(Eli does not) = 1 − 0.17 = 0.83
0.5 G P(Ellis does not) = 1 − 0.68 = 0.32
0.9 0.38 × 0.83 × 0.32 = 0.101 (3 s.f.)
W'
b P(at least one) = 1 – P(none)
0.5 G' = 1 – 0.100 928 = 0.899 (3 s.f.)

21
©HarperCollinsPublishers 2018 Cambridge International AS & A Level Mathematics: Probability & Statistics 1 9780008257767

57767_P014_025.indd 21 7/5/18 3:39 PM


2 PROBABILITY, PERMUTATIONS AND COMBINATIONS

c P(two hit) = 0.62 × 0.17 × 0.32 + 0.62 × 0.83 × 6 a


0.68 + 0.38 × 0.17 × 0.68 1
= 0.428 (3 s.f.) R
1 6
0.428
P(two hit | at least one hits) = = 0.476 (3 s.f.) 3 1
0.899
3 1
4 a x R B
6
1
climbing 3 1
2 G 6
26 35 4 1
R
2 6
1
35 40 3
4
B 1
11 G
1 12
19 3 3
16
1
4 1
R
2 6
bowling table tennis
3
1
b 35 out of 185 do none of the activities. G B
1 12
Therefore the probability of a student 3
doing none of the activities is 35 ÷ 185
= 0.189 (3 s.f.). 1 1 1
b P(B ∩ G) = P(B G) + P(G B) = 12 + 12 = 6
c 19 out of 185 do bowling only.
Therefore the probability of a student doing c
bowling only is 19 ÷ 185 = 0.103 (3 s.f.). 1–3x
R
d P(no activities | not bowling) =
( ) 35
185
1 3

( ) 104
3 1
185 3 1–3x
R B
= 0.336 (3 s.f.) 3
1
3
5 a 1–3x
1–3x G
3
Time taken Time taken R 4x
Total 2 3
 1 min > 1 min
2x 3
Question 1 13 2 B G
2x
right 8 24 3 1 3
3
Question 1 5 1
wrong 8 24 3 2x
x R
2 3
1 3
Total 4 4 1 3
B x
G 3
P(slowly and right) 1
b P(slowly | right ) = 3
P(right)

13
(1 − 3x) + 2x = 1 ⇒ 5 − 15x + 10x = 3
= 24 = 13 3 3 5
()
2
3
16
2
So x =
5

22
©HarperCollinsPublishers 2018 Cambridge International AS & A Level Mathematics: Probability & Statistics 1 9780008257767

57767_P014_025.indd 22 7/5/18 3:39 PM


2
WORKED SOLUTIONS

7 a p = 15, q = 9, r = 7, s = 1 11 The game will be fair if the probability of spinner 1


beating spinner 2 is equal to spinner 2 beating
b P(art | music ) = P(both art and music)
P(music) spinner 1. The same applies between spinner 1 and
spinner 3, and between spinner 2 and spinner 3.
9
= 32 = 9 Probability of spinner 1 beating spinner 2:
( )
16 16
( 63 × 15 ) + ( 62 × 35 ) + ( 16 × 45 ) = 1330
32
=
c P(art) = 24 = 3 ≠ 9 , so these are not
32 4 16 Probability of spinner 2 beating spinner 1:
independent events.
7 15
= ( 15 × 63 ) + ( 15 × 63 ) + ( 15 × 65 ) + ( 15 × 1) = 1730
d × = 0.106 (3 s.f.)
32 31 in 1 versus 3, P(1 win) = 16 , P(3 win) = 14 ,
30 30
 6   3  2  6   3   2  6   3  2 14
8 a       +       +       = 144 in 2 versus 3 P(2 win) =
 2  1   1   1   2  1   1   1   2 25
These probabilities are unequal; therefore the
b 2 × 9! = 725 760 game is unfair.

c 6 × 5 × 7! = 151 200 12 a
0.02 Misshapen (M)

9 a 47 + 62 – 100 = 9 boys both cycle and swim. J


91 = 0.91 = P (student does just one sport). 0.25 0.98 Not misshapen (M′)
b
100
0.03 Misshapen (M)
P(cycling only) 0.45
c P(cycling | one sport only) = K
P(one sport only)
= 0.38 ÷ 0.91 0.97 Not misshapen (M′)
= 0.418 (3 s.f.)
0.3 0.05 Misshapen (M)
d Because some boys do both sports.
L
Alternative answer: P(C ∪ S) not equal to
0.95 Not misshapen (M′)
P(C) + P(S)
e P(C & S) = 9 ÷ 100 = 0.09 ≠ 0.47 × 0.62 b P(M′ ∩ J) = 0.25 × 0.98
10 a Treat the jacks as a block, so you have the
= 0.245
queens, the kings and the jack block, which is
9 things to arrange. There are 4! ways to arrange c P(M) = P(M ∩ J ) + P(M ∩ K) + P(M ∩ L)
the jacks within the jack block. = 0.005 + 0.0135 + 0.015
Total number of arrangements is 12!, so = 0.0335
9! 4! = 0.0182 (3 s.f.). P(K '∩ M )
P(4 jacks together) = d P(K ′ | M) =
12! P(M )
b Ways to arrange the four kings = 4!, and the
= [P( M ∩ J ) + P(M ∩ L)]
other cards = 8! P(M )
So P(2K at one end, 2K at the other)
= 0.02 ÷ 0.0335
4! × 8!
= 12! = 0.00202 (3 s.f.) = 0.597 (3 s.f.)
13 a 5 of the numbers from 2 to 10 are even, so the
c 3 ×  4  4  4 = 288 9
 2  1   1 
answer is 5 .
9
d 288 = 0.582 (3 s.f.)
 12 b Two odd numbers add to give an even
 4  number, but multiply to give an odd number,
so S and T are not mutually exclusive.
c P(S) = P(two cards are both even)
+ P(two cards are both odd)
= 20 × 19 + 16 × 15 = 31
36 35 36 35 63

23
©HarperCollinsPublishers 2018 Cambridge International AS & A Level Mathematics: Probability & Statistics 1 9780008257767

57767_P014_025.indd 23 7/5/18 3:39 PM


2 Probability, permutations and combinations

P(T) = P(two cards are both odd) 17 a T


0.7
16 15 4
= × =
36 35 21 S
4 4 31
P(S ∩ T) = P(T) = ≠ × , so S and T are 0.3
21 21 63 0.35 T'
not independent.
You could argue instead that P(S | T) = 1 ≠ P(S). 0.75 T
14 a 9 × 8 × 7 × 6 × 5 = 15 120 0.4
b Digits that add up to 9 or 18. (1, 2, 6), (1, 3, 5), P
(2, 3, 4), (9, 8, 1), (9, 7, 2), (9, 6, 3), (9, 5, 4), 0.25 T'
(8, 7, 3), (8, 6, 4) are the possible triplets. Each
produce 6 numbers, so 54 in total. 0.25
0.25 T
54 = 3
9 × 8 × 7 = 504, so P(divisible by 9) =
504 28 F
c Each group has 3 members. Number of ways:
 3   3  3 0.75 T'
=       × 3 = 81
 2  1   1 
b P(T ) = 0.35 × 0.7 + 0.4 × 0.75 + 0.25 × 0.25 = 0.608
d The first three cards he picks must add up to a P(F ∩ T ) 0.252
1 c P(F | T ) = P(T )
=
0.6075
= 0.103(3 s.f.)
multiple of 3, so the answer is .
3
18 a
15 a 0.8 − 0.36 = 0.44 R = Red
b 1 − 0.44 = 0.56 1 2 3 4 5 6
c (0.44 × 0.56) + (0.36 × 0.66) + (0.2 × 0.25) 1 1 2 3 4 5 6
= 0.534
2 2 4 6 8 10 12
G = Green

d 1 − 0.534 = 0.466
3 3 6 9 12 15 18
     (0.466)5 = 0.022 (3 d.p.)
4 4 8 12 16 20 24
16 Let us assume that there are 100 students at 5 5 10 15 20 25 30
the school (probabilities are not affected by
6 6 12 18 24 30 36
school size);
x
    b
B G R = Red
1 2 3 4 5 6
C
1 yes yes
26% 31% 2 yes
G = Green

3 yes
4 yes yes
22% 21%
5 yes
6 yes
a 0.26
b 0.21

c P(B ∩ C ) = 0.26 = 0.456(3 s.f.) P(RG is square) = 8 = 2


36 9
P(C ) 0.57
5
d P(G)P(C ) = 52 57 ≠ 31 = P(G ∩ C ) , so not c P(even | square) =
P(even ∩ square) 36 5
= =
100 100 100 P(square) 8 8
independent. 36
5
d P(odd | square) = 1 − = 3
8 8

24
©HarperCollinsPublishers 2018 Cambridge International AS & A Level Mathematics: Probability & Statistics 1 9780008257767

57767_P014_025.indd 24 7/5/18 3:39 PM


2
WORKED SOLUTIONS

 5  24
19 a     = 2760
 3  2 

 5  24  5  24
b
 1  4  +  2  3  = 73 370

20 a 12! = 6 × 11!
2!
10!
b 8 × 4 × 2! = 16 × 10!

c 8 × 8!

d 2
1485

21 a
Red even Red odd Total
Green 1 1 1
even 3 6 2

Green 1 1 1
odd 4 4 2

7 5
Total 12 12 1

b P(product even | sum even) =


P(green even or red even | both odd or both
even)
P(both even)
=
P(both odd or both even)
1
3 4
=
( )
=
1+1 7
3 4

Mathematics in life and work

1
Gearbox Exhaust Expected cost

0.9 Okay 0.855 $0 $0


Okay
0.95 0.1 Fail 0.095 $500 $47.50

0.05 0.9 Okay 0.045 $3000 $135


Fail
0.1 Fail 0.005 $3500 $17.50

2 $200
3 The premium should be $220 per year.

25
©HarperCollinsPublishers 2018 Cambridge International AS & A Level Mathematics: Probability & Statistics 1 9780008257767

57767_P014_025.indd 25 7/5/18 3:39 PM


3 Discrete random variables

3 Discrete random variables


Please note: Full worked solutions are provided as an aid to learning, and represent one approach to answering
the question. In some cases, alternative methods are shown for contrast.
All sample answers have been written by the authors. Cambridge Assessment International Education bears no
responsibility for the example answers to questions taken from its past question papers, which are contained in this
publication.
Non-exact numerical answers should be given correct to 3 significant figures, or 1 decimal place for angles in
degrees, unless a different level of accuracy is specified in the question.
Use graph paper when drawing graphs; graphical solutions given here are not to exact scale but give an indication
of the answer.

Prerequisite knowledge 2 a P(A = a)


1 a {HHH, HHT, HTH, HTT, THH, THT, TTH, TTT} 1
1 36
3
b 1
8 2
18
4 1 1
c = 3
8 2 18
1 1 1 1
2 a + = 4
6 6 3 12
1 ×1 ×2= 2 1
b 5 18
6 6 36
33 1
3 Mean = = 1.1 6
30 9
1
Variance = 77 – 1.12 = 1.36 (3 s.f.) 8
30 18
4 a 6! = 6 × 5 × 4 … = 720 1
9
36
b 10C4 = 10! = 210 1
(6!4!) 10 18
Exercise 3.1A 12
1
9
1 a 1
15
18
x 2 3 4 5 6 7 8 9 10 11 12 1
16
36
1 2 3 4 5 6 5 4 3 2 1 1
P(X = x) 36 36 36 36 36 36 36 36 36 36 36 18
18
1
20
b 18
24 1
18
0.180
1
0.160 25
36
0.140
1
30
0.120 18
P (X = x)

0.100 1
36
0.080 36
0.060
3 a 0.1 + 0.2 + a + 0.25 + 0.3 = 1
0.040
a = 1 – (0.1 + 0.2 + 0.25 + 0.3)
0.020
= 0.15
0.000
2 3 4 5 6 7 8 9 10 11 12 x So P(Y = 1) = 0.15
b P(Y  0) = P(Y = −1) + P(Y = 0)
The distribution is unimodal and symmetrical. = 0.1 + 0.2
26 = 0.3
©HarperCollinsPublishers 2018 Cambridge International AS & A Level Mathematics: Probability & Statistics 1 9780008257767

57767_P026_040.indd 26 7/5/18 3:47 PM


3
WORKED SOLUTIONS

c P(Y > 1) = P(Y = 2) + P(Y = 3) or


= 0.25 + 0.3 P(X  1) = P(X = 1) + P(X = 2) + P(X = 3)
= 0.55 4 2 1
= + +
9 9 27
4 a 19
=
x 2 3 4 5 6 7 8 27

1 1 3 1 3 1 1 7 a
P(X = x) 16 8 16 4 16 8 16
x 1 2 3 4 5 6
b Possible prime numbers are: 2, 3, 5, 7 P(X = x) 1k 2k 3k 4k 5k 6k
Therefore P(X is a prime number) = 1 2 3 =1 4 5 6 =2
P(X = x) 21 21 21 7 21 21 21 7
1 +1+1+1 = 9
or 0.5625
16 8 4 8 16
5 a 
The outcome for each value of x is equal over six
∑p = 1
possible outcomes. k + 2k + 3k + 4k + 5k + 6k = 1
1 21k = 1
Therefore k = .
6 k = 1 (substitute into the table)
21
b P(X < 7) = P(X = 2) + P(X = 3) + P(X = 5)
b P(2  X < 4) = P(X = 2) + P(X = 3)
=1+1+1 =
2 1
+
6 6 6 21 7
1
= or 0.5 5
2 =
21
c P(X  3) = 1 − P(X < 3) c P(even number) = P(X = 2) + P(X = 4) + P(X = 6)
= 1 − P(X = 2)
= 2 + 4 +2=4
=1− 1 21 21 7 7
6 8
5
= r 1 2 3 4
6

2 P(R = r) 0.1r 0.1r 0.1r k


d P(2  X < 11) = 3
P(R = r) 0.1 0.2 0.3 k

6 a There is one way of having 0 blues: G,G,G The value of k = 1 – (0.1 + 0.2 + 0.3) = 1 – 0.6 = 0.4
There are three ways of having 1 blue:
9 
The probability of all the possible outcomes add
G,G,B G,B,G B,G,G
up to 1, therefore:
There are three ways of having 2 blues:
0.4 + 0.25 + 4k + 2k + k = 1
B,B,G B,G,B G,B,B
7k = 0.35
There is one way of having 3 blues: B,B,B
k = 0.05
x 0 1 2 3
The probability that at least two under graduates

( 23 ) () () ()
3
3
1 2
2
1
2
2 1 are late is:
P(X = x) 3× × 3× × 3
3 3 3 3 P(X  2) = 1 – P(X  1) = 1 – (0.4 + 0.25) = 0.35

8 4 2 1 10 a T
 he probability of all the possible outcomes add
P(X = x) 27 9 9 27 up to 1, therefore:
0 + 3r + 9r + 4r + 3r + r = 1
b P(X  1) = 1 − P(X = 0)
20r = 1
=1− 8
27 r = 0.05
19
=
27

27
©HarperCollinsPublishers 2018 Cambridge International AS & A Level Mathematics: Probability & Statistics 1 9780008257767

57767_P026_040.indd 27 7/5/18 3:47 PM


3 Discrete random variables

b
c 0 1 2 3 4 5
P(C2 = c) 0.25k 1.25k 2.25k 3.25k 3.5k 4.5k

P(C2 = c) 1 1 1 5 1 1 9 1 3 13 1 13 7 1 7 9 1 3
× = × = × = × = × = × =
4 15 60 4 15 12 4 15 20 4 15 60 2 15 30 2 15 10

0.25k + 1.25k + 2.25k + 3.25k + 3.5k + 4.5k = 1


1
15k = 1 ⇒ k =
15
1 59
P(C 2  1) = 1 − P(C 2 = 0) = 1 − =
60 60

Exercise 3.2A 4 a i 0.2 + 0.15 + 0.35 + a + 0.1 = 1


a = 1 − (0.2 + 0.15 + 0.35 + 0.1)
1 a E(X) = −2 × 0.13 − 1 × 0.27 + 0 × 0.1 + 1 × 0.18
+ 2 × 0.22 + 3 × 0.1 = 0.2

= 0.39 ii E(X) = 0
 × 0.2 + 1 × 0.15 + 2 × 0.35 + 3 × 0.2
+ 4 × 0.1
b E(Y) = 2 × 1 + 4 × 5 + 6 × 1 + 8 × 1
12 12 3 6  = 1.85
31
= = 5.17 (3 s.f.) iii
6
2 a E(Z) = 5 × 0.4 + 6 × 0.3 + 7 × 0.2 + 8 × 0.1 x2 0 1 4 9 16
=6 P(X = x) 0.2 0.15 0.35 0.2 0.1
b E(X2) = 0 × 0.2 + 1 × 0.15 + 4 × 0.35 + 9 × 0.2
z2 25 36 49 64 + 16 × 0.1
P(Z = z) 0.4 0.3 0.2 0.1 = 4.95
E(Z 2) = 25 × 0.4 + 36 × 0.3 + 49 × 0.2 + 64 × 0.1 b i k + 2k + 3k + k = 1
= 37 7k = 1
c (E(Z))2 ≠ E(Z 2) since (E(Z))2 = 62 = 36 ≠ 37 = E(Z 2). k = 1 = 1.14 (3 s.f.)
7
1 2 3 1
ii E(Y ) = −2 × 7 + 0 × + 2 × 7 + 4 × 7
3 1 2 3 4 5 6 7
8
=
1 0 1 2 3 4 5 7
1 2 3 1
2 1 0 1 2 3 4 2
iii E(Y ) = 4 × 7 + 0 × 7 + 4 × 7 + 16 × 7
3 2 1 0 1 2 3 32
= = 4.57 (3 s.f.)
7
4 3 2 1 0 1 2
5 4 3 2 1 0 1 5 a x P(X = x)
6 5 4 3 2 1 0
1
7
= 0.14
50
a
22
2 = 0.44
d 0 1 2 3 4 5 50
18
1 5 2 1 1 1 3 = 0.36
P(D = d) 6 18 9 6 9 18
50
1
4 50 = 0.02
1 5 2 1
b E(D) = 0 × +1× +2× +3× 1 +4×
6 18 9 6 9 2
1 5 50 = 0.04
+5×
18
35 b E(X) = 1 × 0.14 + 2 × 0.44 + 3 × 0.36 + 4 × 0.02
= = 1.94 (3 s.f.)
18 + 5 × 0.04
= 2.38

28
©HarperCollinsPublishers 2018 Cambridge International AS & A Level Mathematics: Probability & Statistics 1 9780008257767

57767_P026_040.indd 28 7/5/18 3:47 PM


3
WORKED SOLUTIONS

c P(X > 2.38) = P(X = 3) + P(X = 4) + P(X = 5) 1 1 1 4 1


b E(X) = 1 × + 2 × + 3 × + 4 × + 5 ×
10 8 8 10 8
= 0.36 + 0.02 + 0.04
+6× = 1 37 or 3.7
= 0.42 8 10
6 x P(X = x) c x 1 2 3 4 5 6
0 0.73 = 0.343
2x 2 4 6 8 10 12
1 3 × (0.72)(0.3) = 0.441
2 3 × (0.7)(0.32) = 0.189 1 1 1 4 1 1
P(X = x) 10 8 8 10 8 8
3 0.33 = 0.027

E(X) = 0 × 0.343 + 1 × 0.441 + 2 × 0.189 + 3 × 0.027 E(2X) = 2 × 1 + 4 × 1 + 6 × 1 + 8 × 4 + 10 × 1


10 8 8 10 8
= 0.9 1
+ 12 × = 7.4
8
7 a + 4b + 2b + a = 1
Alternative method:
2a + 6b = 1 (1) 37
E(2X) = 2 E(X) = 10 × 2 = 7.4
−1 × a + 0 × 4b + 1 × 2b + 2 × a = 5
12
10 a
5 y 2 4 6 8
−a + 2b + 2a =
12
5 2 4 6 8
a + 2b = P(Y = y) 20 20 20 20
12
12a + 24b = 5 (2) b
y 2 4 6 8
8a + 24b = 4 (1) × 4
y–1 2–1 4–1 6–1 8–1
4a = 1 (2) – 4(1)
2 4 6 8
a=
1 P(Y = y) 20 20 20 20
4

()
Substitute a into (1): 2 1 + 6b = 1
4
E (Y–1) = 2
−1
×
2
20
+ 4 −1 ×
4
20
+ 6 −1 ×
6
20
8 1
1 + 8 −1 × =
6b = 1 − 20 5
2
1 Exercise 3.3A
=
2
1 1 a Var(X) = (−2)2 × 0.13 + (−1)2 × 0.27 + 02 × 0.1
b= ÷6
2 + 12 × 0.18 + 22 × 0.22 + 32 × 0.1 – 0.092
1
= = 2.60
12 2
+ 62 × 3 + 82 × 6 −  
1 5 1 1 31
b Var(Y) = 22 × + 42 ×
8 z 4 12 12 12  6
P(Z = z) x 1−x
 =
107
= 2.97
36
4x + 12(1 − x) = 7
4x + 12 − 12x = 7 2 d 1 2 3 4 5 6
5 = 8x 1 1 1 1 1 1
5 P(D = d ) 6 6 6 6 6 6
x = = 0.625
8
z 4 12 1 1 1 1 1 1
E(D) = 1 × +2× +3× +4× +5× +6×
6 6 6 6 6 6
P(Z = z) 0.625 0.375
= 3.5
1 1 1 1 1
9 a ∑p = 1  2×
Var(D) = 1
6
+ 22 × +32 × + 42 × + 52 ×
6 6 6 6
k + 4k + 4 = 1 1
+ 62 × – 3.52
8 6
k= 1 35
10 = = 2.92
12
29
©HarperCollinsPublishers 2018 Cambridge International AS & A Level Mathematics: Probability & Statistics 1 9780008257767

57767_P026_040.indd 29 7/5/18 3:47 PM


3 Discrete random variables

3 a 1 2 3 4 6 a E(A) = 10 × 0.6 + 25 × 0.25 + −5 × 0.15


= 11.5%
1 2 3 4 5
E(B) = 10 × 0.55 + 50 × 0.25 + −30 × 0.2
2 3 4 5 6
= 12%
3 4 5 6 7
4 5 6 7 8  02 × 0.6 + 252 × 0.25 + (−5)2 × 0.15
b Var(A) = 1
– 11.52
s 2 3 4 5 6 7 8 = 87.75
1 2 3 4 3 2 1  02 × 0.55 + 502 × 0.25
Var(B) = 1
P(S = s) 16 16 16 16 16 16 16 + (−30)2 × 0.2 − 122

1 2 3 4 = 716
b E(S) = 2
 × 16 + 3 × 16 + 4 × 16 + 5 × 16
c The expectation of stock B is higher, so on
3 2 1 average, he would get better returns on
+ 6 × 16 + 7 × 16 + 8 × 16
his investment. The variance of B is much
=5
1 2 3 4 higher so there is more risk. The stockbroker
Var(S) = 22 × 16 + 32 × 16 + 42 × 16 + 52 × 16 may make a higher profit in stock B but
3 2 1 could also lose a lot more than in stock A.
+ 62 × 16 + 72 × 16 + 82 × 16 − 52
7 a P(R = 3) means having two rolls that are not 1
= 2.5
and then a third roll that is a 1, or 3 rolls that are
4 a 0.1 + p + 0.2 + 0.35 = 1
not 1.
p = 1 − (0.1 + 0.2 + 0.35) 2 3
= 0.35 =  5  × 1 +  5 
 6 6  6
b 2 × 0.1 + 5 × 0.35 + a × 0.2 + 8 × 0.35 = 5.95
25
4.75 + 0.2a = 5.95 =
36
0.2a = 5.95 − 4.75
b r 1 2 3
= 1.2
1.2 1 5× 1= 5 25
a= P(R = r) 6 6 6 36 36
0.2
=6 1 5 25
c E(R) = 1 × +2× +3×
 2 × 0.1 + 52 × 0.35 + 62 × 0.2
c Var(X) = 2 6 36 36
+ 82 × 0.35 − 5.952 91
= = 2.53
36
= 3.35
( )
2
1 5 25 91
5 a a + 3b + 2a + b = 1 Var(R) = 12 × + 22 × + 32 × –
6 36 36 36
3a + 4b = 1 (1) = 0.583
1 × a + 3 × 3b + 5 × 2a + 7 × b = 3.8 8 a Let G be the number of girls who are appointed.
a + 9b + 10a + 7b = 3.8
P(G = 0) = 8 × 7 = 14
11a + 16b = 3.8 (2) 20 19 95

12a + 16b = 4 (1) × 4 P(G = 1) = 8 × 12 + 12 × 8 = 48


20 19 20 19 95
Subtracting: P(G = 2) = 12 × 11 = 33
20 19 95
a = 0.2
Substitute into (1): g 0 1 2
3(0.2) + 4b = 1
14 48 33
4b = 0.4 P(G = g ) 95 95 95
b = 0.1
 2 × 0.2 + 32 × 0.3 + 52 × 0.4 + 72
b Var(Y) = 1 b E(G) = 0 × 14 + 1 × 48 + 2 × 33 = 6 = 1.2
95 95 95 5
× 0.1 – 3.82
Rounding to the nearest integer, one girl is
 = 3.36 expected to be appointed.

30
©HarperCollinsPublishers 2018 Cambridge International AS & A Level Mathematics: Probability & Statistics 1 9780008257767

57767_P026_040.indd 30 7/5/18 3:47 PM


3
WORKED SOLUTIONS

9 Let A be the profit from new car Model A: 0 5


 5
P(H = 0) =    1   1  = 1
a 3000 2500 –500  0  2   2  32

P(A = a) 0.5 0.3 0.2 P(H = 3) = 1 × 1 + 5 × 1 = 7


8 3 16 3 48
E(A) = 3000 × 0.5 + 2500 × 0.3 + (−500) × 0.2 = 2150
b
Let B be the profit from new car Model B: h P(H = h) P(H = h)
b 3500 3000 –1000
0 ( 21 + 81 + 321 ) × 31 7
32
P(B = b) 0.4 0.4 0.2

E(B) = 3500 × 0.4 + 3000 × 0.4 + (−1000) × 0.2 = 2400 1 ( 21 + 83 + 325 ) × 31 11


32
E(B) > E (A), so the finance director would support
new car Model B.
10 a Let X be the number showing on the spinner:
2 ( 83 + 165 ) × 31 11
48

x 1 3 5 3 ( 81 + 165 ) × 31 7
48
1 1 1
P(X = x) 3 3 3 5 ×1 5
4 32 3 96
If the spinner shows '1':
1 ×1 1
5
P(H = 1) = 1 32 3 96
2
P(H = 0) = 1 − 1 = 1 7 + 1 × 11 + 2 × 11 + 3 × 7 + 4 × 5
2 2 c E(H) = 0 ×
32 32 48 48 96
If the spinner shows '3':
+5× 1 3
= or 1.5
 3 1 2 96 2
P(H = 1) =    1   1  = 3
 1  2   2  8
Var (H) =  0 2 × 7 + 12 × 11 + 22 × 11 + 32 × 7 + 4 2 × 5 + 52 × 1  −
 32 32 48 48 96 96 
 3  1  2  1  1 3 2
P(H = 2) =  0 2 × 7 + 12 × 11 + 22 × 11 + 32 × 7 + 4 2 × 5 + 52 × 1  −  3 
 2  2   2  = 8  32 32 48 48 96 96   2 
3 0
 3 = 15 or 1.42
P(H = 3) =    1   1  = 1 12
 3  2   2  8

P(H = 0) = 1 − 3 − 3 − 1 = 1
Exercise 3.4A
8 8 8 8
1 a i P(X = 2) = (1 − 0.2) × 0.2 = 0.16
If the spinner shows '5': ii P(X = 5) = (1 − 0.2)4 × 0.2 = 0.081 92
1 4 iii P(X = 10) = (1 − 0.2)9 × 0.2 = 0.027 (3 d.p.)
 5
P(H = 1) =    1   1  = 5 b i P(X = 2) = (1 − 0.8) × 0.8 = 0.16
 1  2   2  32
ii P(X = 5) = (1 − 0.8)4 × 0.8 = 0.001 28
 5  1  2  1  3 5 iii P(X < 3) = P(X = 1) + P(X = 2)
P(H = 2) =
 2  2   2  = 16 = 0.8 + (1 − 0.8) × 0.8
3 2 = 0.8 + 0.16 = 0.96
 5
P(H = 3) =    1   1  = 5
( 78 ) ( 18 ) = 0.084 (3 d.p.)
3
 3  2   2  16 c i P(X = 4) =

 5 4 1 ii P(X  4) = P(X = 1) + P(X = 2) + P(X = 3) + P(X = 4)


P(H = 4) =    1   1  = 5
   
( )( ) ( ) ( 18 ) + ( 78 ) ( 18 )
2 3
 4 2 2 32 1 7 1 7
= + 8 8 + 8
8
5 0
 5
P(H = 5) =    1   1  = 1 = 0.414 (3 d.p.)
 5  2   2  32
iii P(3  X < 7) = P(X = 3) + P(X = 4) + P(X = 5)
+ P(X = 6)
31
©HarperCollinsPublishers 2018 Cambridge International AS & A Level Mathematics: Probability & Statistics 1 9780008257767

57767_P026_040.indd 31 7/5/18 3:47 PM


3 Discrete random variables

( ) ( 18 ) + ( 78 ) ( 18 ) + ( 78 ) ( 18 ) b P(X = n) = (1 − s)n − 1s
2 3 4
7
= 8
c P(X = 2) = 0.21

+ (8) (8)
5
7 1 (1 − s)s = 0.21
s − s2 = 0.21
= 0.317 (3 d.p.)
s2 − s + 0.21 = 0
d i P(X < 4) = P(X = 1) + P(X = 2) + P(X = 3)
s = 0.7 or 0.3
= 0.4 + 0.6(0.4) + 0.62(0.4)
Since the probability of a person answering
= 0.784 the phone on the first call is greater than 0.5,
ii P(X = 5) = 0.64 × 0.4 = 0.051 84 s = 0.7.
iii P(X > 5) = (1 − 0.4)5 = 0.077 76
9 a X ~ Geo  1 
e i P(X  3) = P(X = 1) + P(X = 2) + P(X =3)  12 

(1211 )
19
= 0.16 + (1 − 0.16)(0.16)
P ( X  20 ) = P(X > 19) = = 0.191
+ (1 − 0.16)2(0.16) = 0.407 (3 d.p.)
ii P(X > 6) = (1 − 0.16)6 = 0.351 (3 d.p.) b P(X > x) = 0.1
x
iii P(X  4) = P(X > 3) = (1 − 0.16)3 = 0.593 (3 d.p.)  11  = 0.1
 12 
2 X ~ Geo (101 ) x ln
11
12
= ln0.1
5
P(X = 6) =  1 − 1   1  = 0.0590 (3 s.f.) x = 26.46
 10   10 
3 X ~ Geo(0.5) She must collect at least 27 beetles.
P(X = 4) = 0.53 × 0.5 = 0.0625 10 a Y ~ Geo  1 
4 a X ~ Geo(0.25)  20 

( 1920 ) ( 201 ) = 0.031 5


9
P(X < 5) = P(X = 1) + P(X = 2) + P(X = 3) + P(X = 4) P (Y = 10 ) =
 .25 + 0.75 × 0.25 + 0.752 × 0.25
=0
b P (Y > 25) = ( 19 ) = 0.277
25
+ 0.753 × 0.25
20
= 0.684
c P (Y  15) = 1 − ( 19 ) = 0.537
15
b P(X  7) = P(X > 6) = (1 − 0.25)6 = 0.178
20
5 a X ~ Geo(0.35)
P(X = 6) = 0.655 × 0.35 Exercise 3.4B
= 0.0406 (3 s.f.)  10
b P(X  10) = P(X > 9) 1 a i P(X = 2) =   × 0.22 × 0.88 = 0.302
 2
= 0.659 = 0.0207 (3 s.f.)
 10
6 X ~ Geo(0.4) ii P(X = 5) =   × 0.25 × 0.85 = 0.0264
 5
There is time for three attempts in 1 h 30 min
(90 ÷ 25 = 3.6)  10
iii P(X = 10) =   × 0.210 × 0.80 = 1.02 × 10−7
P(X  3) = P(X = 1) + P(X = 2) + P(X = 3)  10
= 0.4 + 0.6 × 0.4 + 0.62 × 0.4 = 0.784  7
b i P(X = 2) = × 0.82 × 0.25 = 0.004 30
()
 2
1
7 a X ~ Geo 5
 7
P(X = 4) = ( ) ( ) = 0.1024
4 1
3 ii P(X = 5) = × 0.85 × 0.22 = 0.275
 5
5 5

b P(X > 5) = ( ) = 0.328 (3 s.f.)


5 iii Since the binomial distribution is discrete,
4
5 P(X < 3) = P(X = 0) + P(X = 1) + P(X = 2)
8 a Each phone call is independent.  7  7
= × 0.80 × 0.27 +   × 0.81 × 0.26
The probability of the phone calls getting  0  1
answered is constant.
 7
+   × 0.82 × 0.25 = 0.004 67
 2
32
©HarperCollinsPublishers 2018 Cambridge International AS & A Level Mathematics: Probability & Statistics 1 9780008257767

57767_P026_040.indd 32 7/5/18 3:47 PM


3
WORKED SOLUTIONS

 20  17
c i P(X = 18) =   × 0.12518 × 0.8752 e i P(X  2) =   × 0.160 × 0.8417
 18   0
 = 8.08 × 10−15  17
+   × 0.161 × 0.8416
ii From cumulative binomial probabilities,  1
P(X  3) = P
 (X = 0) + P(X = 1) + P(X = 2)  17
+   × 0.162 × 0.8415
+ P(X = 3)  2
 20
( ) × ( 78 )
0 20
1 = 0.473
= ×
0 8
 17
P(X = 5, 6, 7) =   × 0.165 × 0.8412
ii

( 78 ) ( ) × ( 78 ) () () ( ) ×× ( 7878)
20 11 20
()
 5
19 22 18 11
19
 20 1
2 18
 20 1
1 × 77 19
 20 18
 20
11
× +  × +  × × 8 ++   ×× 8 ++   ××  17
 2 8  1 8 8  22  8 88 17
 11 
  +   × 0.166 × 0.8411 +   × 0.167 × 0.8410
+  ×( ) × ( )
 20 3 17  6  7
1 7
 3 8 8        = 0.120

= 0.765 iii P(2 < X  15) = P(X  15) − P(X  2)


iii P(3 < X  18) =  P(X  18) − P(X  3)   17 16 1  17  17 0
= (1 − P(X = 19 or 20)) − 0.765 33 = 1 −   16 × 0.16 × 0.84 +  17 × 0.16 × 0.84 
  20
() ()  20 1 
()
19 1 20
1 7
1 −    × 8 × 8 +   × 8  17   17  
  19   20   × 0.1616 × 0.841 +   × 0.1617 × 0.840  − 0.473…
  16   17  
− 0.765 33
= 0.527
= 0.235
d i Since the binomial distribution is discrete, 2 Number of trials is eight, therefore n = 8.
1
P(X < 4) = 1 − P(X = 4 or 5) Probability of rolling a six is 1 , therefore p = .
6 6
  5 5 0  5 4 1
 Since there are a set number of independent
=1−
  × 0.4 × 0.6 +  4 × 0.4 × 0.6 
  5  trials with a consistent probability, the problem
= 0.913 can be modelled using a binomial distribution.
Let X be the number of sixes rolled.
Alternatively, P(X = 0, 1, 2 or 3)
 1
Therefore, X ~ B 8, 
 6
 5
=   × 0.40 × 0.65 +
5
( ) × ( 65 )
× 0.41 × 0.64  8 3 5
 0  1 P(X = 3) =   × 1 = 0.104
 3 6
 5
× 0.42 × 0.63 +   × 0.43 × 0.62
5
+  10
 2  3 3 P(X = 6) = × (s)6 × (1 − s)4
 6 
= 0.913
 5 4 a P(X < 3) = P(X  2)
ii P(X = 5) =   × 0.45 × 0.60 = 0.010 2
 5  12 0 12  12 1 11  12
=   × 0.25 × 0.75 +   × 0.25 × 0.75 +   × 0.
iii P(0  X < 3)  0  1  2
= P(X < 3)  12
= P(X  2) = P(X = 0) + P(X = 1) + P(X = 2) +   × 0.251 × 0.7511
 1
 5
=   × 0.40 × 0.65  12
 0 +   × 0.252 × 0.7510
 2
 5
+ × 0.41 × 0.64 = 0.391
 1
 12
 5 b P(X  9) =   × 0.259 × 0.753
+   × 0.42 × 0.63  9
 2
 12
= 0.683 +   × 0.2510 × 0.752
 10

33
©HarperCollinsPublishers 2018 Cambridge International AS & A Level Mathematics: Probability & Statistics 1 9780008257767

57767_P026_040.indd 33 7/5/18 3:47 PM


3 Discrete random variables

 12 Therefore, X ~ B(10, 0.25)


+ × 0.2511 × 0.751
 11  10
P(X = 3) =   × (0.25)3 × (0.75)7
 3
 12
+   × 0.2512 = 0.250 (3 s.f.)
 12
b P(X  3) = 1 − P(X  2)
= 0.000 392 (3 s.f.)
= 1 − 0.5256
5 a 
Number of students selected is 30, therefore = 0.474 (3 s.f.)
n = 30.
7 a Number of students is 20, therefore n = 20.
Probability of a student asking for paper is 3
Probability of having a birthday in January is
out of 5, therefore p = 0.6.
1 , therefore p = 1 .
Since there is a set number of independent 12 12
trials and a fixed probability of success, the Since there is a set number of independent
problem can be modelled as a binomial students and a fixed probability of having
distribution. a birthday in January, the problem can be
Let X be the number of students who ask for modelled as a binomial distribution.
paper. Let X be the number of students with
Therefore, X ~ B(30, 0.6) birthdays in January.
Therefore, X ~ B 20, 
1
 30  12 
P(X = 10) =   × (0.6)10 × (0.4)20
 10 
P(X = 1) =   × 1
20
( ) × (1211 )
1 19
= 0.002 00 (3 s.f.) = 0.319 (3 s.f.)
 1  12
b P(X < 25) = 1 − P(X  25)
b P(X  4) = 0.978 (3 s.f.)
  30 25 5  30  26 4  30   30   30  30 
= 1 –    × 0.6 × 0.4 +   × 0.6 × 0.48 + a   × 0.6 27of
Number
 .43 +   chosen
× 0customers × 0.6 28 ×is05, 4 2therefore
.4 +   × 0.6 29 × 0.4 +   × 0.630 
  25  26  27  28  29  30 
n = 5. Probability of being in the 18–25 age
  30 group is 0.4. Since there is a set
 30number of
25 5  30  26 4  30  27 3  30  28  30  
  25 × 0.6 × 0.4 +  26 × 0.6 × 0.4 +  27 × 0.6 × 0.4 +  28 × 0.6independent 4 2 +  customers
× 0.4 × 0.6 29
× 0.4 + × 0.6 30

 29  30
and a fixed probability,
        
the problem can be modelled as a binomial
 30   30   30   30 distribution 30B(5, 0.4).
0.6 25 × 0.45 +   × 0.6 26 × 0.4 4 +   × 0.6 27 × 0.43 +   × 0.6 28 × 0.44 2 +   × 0.6 29 × 0.4 +   × 0.630 
 26  27  28  29 Let X be the 30number of customers in the age
group 18–25. Therefore:
26 4  30  27 3  30  28 2  30  29  30 
30P(X < 2) = P(X = 0) + P(X = 1)
0.6 × 0.4 +   × 0.6 × 0.4 +   × 0.6 × 0.4 4 +   × 0.6 × 0.4 +   × 0.6 
 27  28  29  30 
 5 5  5
=   ( 0.4 ) ( 0.6 ) +   ( 0.4 ) ( 0.6 )
0 1 4
 30   30   30    0  1
0.6 27 × 0.43 +   × 0.6 28 × 0.4
4 2 +   × 0.6 29 × 0.4 +   × 0.630 
 28  29  30 
= 0.337
 30  b The binomial distribution will not be
+   × 0.630 
 30  suitable. Customers chosen from the same
= 1 − 0.005 659 = 0.994 (3 s.f.) table are not independent of each other;
6 a 
Number of flights in the winter is 10, therefore they are more likely to be in the same age
n = 10. group.
9 A packet contains 10 gingerbread men biscuits,
Probability of poor visibility is 25%,
therefore n = 10. Probability of being broken is
therefore p = 0.25.
0.03. Since there are a set of independent trials
Since there are a set number of independent with a fixed probability, this can be modelled as a
trials with a fixed probability of success, binomial distribution, X~B (10, 0.03).
the problem can be modelled as a binomial
P(X = 2) =   ( 0.03)2 ( 0.97 )8 = 0.031 7
distribution. 10
 2 
Let X be the number of poor visibility
encounters.

34
©HarperCollinsPublishers 2018 Cambridge International AS & A Level Mathematics: Probability & Statistics 1 9780008257767

57767_P026_040.indd 34 7/5/18 3:47 PM


3
WORKED SOLUTIONS

10 A box contains 10 duck eggs, therefore n = 10. ii Var(X) = np(1 − p)


Probability of any egg being broken is 0.1. Since = 5 × 0.1 × (1 − 0.1)
there is a set number of independent duck eggs
= 0.45

()
and a fixed probability, this problem can be
1
modelled as a binomial distribution, X~B (10, 0.1). 3 a X ~ Geo
6
P(X  2) = 1 – P(X  1)
E(X ) = 1 = 1 = 6
  10 10    10  9  p  1
 0  ( 0.1) ( 0.9)  +   1  ( 0.1) ( 0.9)  
0 1
= 1 –   6
    
 1
  10 b Y ~ B 42, 
10    10 9   6
   ( 0.1) ( 0.9)  +    ( 0.1) ( 0.9)  
0 1

  1    1   E(Y) = np
1
= 1 – 0.736 1 = 42 ×
6
= 0.264 =7

Exercise 3.5A 4 a E(X) = np


= 30 × 0.1
1= 1
1 a E(X) =
p 0.3 =3
Var(X) = np(1 − p)
= 10
3 = 30 × 0.1 × (1 − 0.1)
= 3.33 (3 s.f.) = 2.7
1 1 b P(μ  X < μ + σ) = P(3  X < 3 + 1.643)
b E(X ) = =
p 0.45
= P(X = 3) + P(X = 4)
20
=  30
9 = × 0.13 × 0.927
 3 
= 2.22 (3 s.f.)
1 1  30
c E(X) = =
()
+ × 0.14 × 0.926
p 3  4 
7
= 0.413
7
= 5 E(X) = np
3
= 2.33 (3 s.f.) = 25 × 0.6
= 15
2 a i E(X) = np Var(X) = np(1 − p)
= 10 × 0.2 = 2 = 25 × 0.6 × (1 − 0.6)
ii Var(X) = np(1 − p) =6
= 10 × 0.2 × (1 − 0.2) So σ = 6 = 2.449 4… = 2.45 (3 s.f.)
P(μ − σ < X < μ + σ) = P(15 − 2.449 5 < X < 15 + 2.449 5)
= 1.6
= P(12.550 5 < X <17.449 5)
b i E(X) = np = P(X = 13) + P(X = 14) + P(X = 15)
= 150 × 0.3 = 45 + P(X = 16) + P(X = 17)
ii Var(X) = np(1 − p)  25
=   × 0.613 × 0.412
= 150 × 0.3 × (1 − 0.3)  13 
= 31.5  25
+   × 0.614 × 0.411
c i E(X) = np  14 
3
= 20 × = 7.5  25
8 +   × 0.615 × 0.410
ii Var(X) = np(1 − p)  15 
3
= 20 × × (1 − 3 )  25
8 8 +   × 0.616 × 0.49
 16 
= 4.687 5
 25
d i E(X) = np +   × 0.617 × 0.48
 17 
= 5 × 0.1 = 0.5
= 0.693 (3 s.f.)
35
©HarperCollinsPublishers 2018 Cambridge International AS & A Level Mathematics: Probability & Statistics 1 9780008257767

57767_P026_040.indd 35 7/5/18 3:48 PM


3 Discrete random variables

6 E(X) = np = 2.5 (1) b To obtain the second $1 coin, there are only 14
Var(X) = np(1 − p) = 1.875 (2) coins in the bag. Therefore the probability of
getting a $1 coin is 4 . Let Y be the number of
( 2) : 1 − p = 1.875 14
(1) extra selections; this again can be modelled as
( )
2.5
geometrical distribution Y~Geo 4 .
1 − p = 0.75 14
p = 0.25, and by substitution, n = 10 1 = 7 or 3.5 selections
E(X) =
2.5 4 2
Or E(X) = np = 2.5 so n = p (1) 14
Var(X) = np(1 − p) = 1.875 (2) 10 The probability of getting a chocolate egg with a
1
limited edition toy is . Let X be the number of
Substitute (1) into (2):  2.5  p(1 − p) = 1.875 12
 p  chocolate eggs up to and including the first limited
2.5(1 − p) =1.875 edition toy, this problem can be modelled as

1−p=
1.875
2.5
geometrical distribution, X ∼ Geo
1
12
.( )
= 0.75 E(X) = 1 = 12 eggs
1
p = 0.25, n = 10 12
7 (
a X ~ B n, 13
20 ) Expected cost: $1.20 × 12 = $14.40

E(X) = 39 Exam-style questions


13 1 a 0.3 + P(X = 1) + 0.4 + 0.05 = 1
n× = 39
20
20 P(X = 1) = 1 – (0.3 + 0.4 + 0.05)
n = 39 ×
13 = 0.25
= 60
39 21
 60 b E(X) = 0 × 0.3 + 1 × 0.25 + 2 × 0.4 + 3 × 0.05
b P(X = 39) =   ×  13  ×  7 
 39  20   20  = 1.2
= 0.107 (3 s.f.)  20
2 a P(T = 3) =   × 0.43 × 0.617
8 a X ~ B(8, 0.1)  3
E(X) = 8 × 0.1 = 0.8, so about 1 plate is expected to = 0.012 3
be damaged in each box. b P(T  3) = P(T = 0) + P(T = 1) + P(T = 2) + P(T = 3)
b P(X  1) = 1 − P(X = 0)  20
=   × 0.40 × 0.620
  8   0
= 1 −    × 0.10 × 0.98 
  0   20
+   × 0.41 × 0.619
= 0.570 (3 s.f.) 1

+   × 0.41 × 0.618
c Y ~ B(40, 0.569 5…) 20
 2 
E(Y) = 40 × 0.569 5…
= 22.8 (3 s.f.), so 23 boxes are expected to
+  20 × 0.43 × 0.617
contain one or more damaged plates.  3 
d There can only be an integer number of boxes. = 0.016 0
9 a There are 15 coins in total, five of them c E(T) = 20 × 0.4
are $1 coins. Therefore the probability of =8
5 Var(T) = 20 × 0.4 × 0.6
selecting one $1 coin is . Let X be the
15
number of selections up to and including = 4.8
the first $1 coin, this can be modelled as 3 a The trials need to be independent of each other.
geometrical distribution X~Geo 5 .
15 ( ) The probability of a success needs to be constant.
1 = 3 selections There must be only two possible outcomes
E(X) =
5 with fixed probabilities.
15
36
©HarperCollinsPublishers 2018 Cambridge International AS & A Level Mathematics: Probability & Statistics 1 9780008257767

57767_P026_040.indd 36 7/5/18 3:48 PM


3
WORKED SOLUTIONS

b P(X = 3) = (1 − 0.4)2 × 0.4  15


b P(X = 15) =   (0.1)15(0.9)0 = 1 × 10−15
= 0.144  15
c P(X > 3) = (1 − 0.4)3 c E(X) = 15 × 0.1 = 1.5
= 0.216
7 a P(3 fails, then 1 success) = 0.73 × 0.3 = 0.103
4 a If S = 4 then you can have 1 and 3 or 3 and 1.
b P(1st success after 4th throw) = 0.74 = 0.240
so P(S = 4) = 1 × 2 + 2 × 1
4 3 4 3 c P(2nd success on 4th throw) = P(1 success in
1 1st 3 throws, then 1 success on 4th throw)
=
3
  3 
b P(S = 1) = 2 ×
1 1
4 3 ( ) =   (0.3)1(0.7)2  × 0.3
  1  

P(S = 3) = 2 ( × )
2 1 = 0.132
4 3 d P(Lucas first scores on his 2nd shot)
P(S = 6) = 2 1 × 1 
= 0.7 × 0.8 × 0.3 = 0.168
 4 3
e P(Stefan first scores on his 7th attempt)
s 1 3 4 6 = (0.7 × 0.8)6 × 0.7 × 0.2 = 0.00432
8 a E(R) = −2a + 3(1−a)
1 1 1 1
P(S = s) 6 3 3 6 = 3 – 5a
1 1 1 1 Var(R) = (−2)2a + 32(1 – a) – E2(R)
c E(S) = 1 × 6 + 3 × 3 + 4 × 3 + 6 × 6
= 4a + 9(1 – a) – (3 – 5a)2
= 3.5
= 25a – 25a2
1 1 1 1
Var(S) = 12 × 6 + 32 × 3 + 42 × 3 + 36 × 6 – 3.52 = 25a(1 – a)
= 2.25 b E(R) = 2
5 X ~ B(10, 0.21) 1
Therefore: 3 – 5a = 2 ⇒ a=
5
 10
a P(X = 2) =   (0.21)2 (0.79)8 = 0.301 (3 s.f.)
 2 Var(R) = 25 × 4 = 4 ⇒ σ = 4=2
5 5
b P(X  4) = 1 – P(X  3) 9 Y ~ B(30, 0.05)
= 1 – [P(X = 0) + P(X = 1) + P(X = 2) + P(X = 3)]  30
a P(X = 2) =   (0.05)2(0.95)28 = 0.259
 10  10   10   10   2 
= 1 −   (0.21)0(0.79)10 +   (0.21)1(0.79)9 +   (0.21)2(0.79)8 +   (0.21)3(0.79)7 
 0   1  2 b P(X > 3 3) = 1 − P(X  3)

 10  10  10  = 1 − [P(X = 0) + P(X = 1) + P(X = 2)


0.21)0(0.79)10 +   (0.21)1(0.79)9 +   (0.21)2(0.79)8 +   (0.21)3(0.79)7  + P(X = 3)]
 1  2  3 
= 1 − 0.860 8  30  30  30
= 1 −   ( 0.05) ( 0.95) +   ( 0.05) ( 0.95) +   (
0 30 1 29
= 0.139 (3 s.f.)   0   1   2
c P(4 < X < 9) = P
 (X = 5)+ P(X = 6) + P(X = 7)
 30 30  30  29  30  28 
= 1 −   ( 0.05) ( 0.95) +   ( 0.05) ( 0.95) +   ( 0.05) ( 0.95) 
+ P(X = 8) 0 1 2

 0 
  10  1   2  
 10  10  10 
=   (0.21)5(0.79)5 +   (0.21)6(0.79)4 +   (0.21)7(0.79)3 +   (0.21)8(0.79)2 
 5   30 6  0 30  30 7   8 
29  30 28  30  
= 1 −   ( 0.05) ( 0.95) +   ( 0.05) ( 0.95) +   ( 0.05) ( 0.95) +   (0.05)3 (0.95)27 
1 2

 10  10  10   10


0    1  2  
3 
=   (0.21)5(0.79)5 +   (0.21)6(0.79)4 +   (0.21)7(0.79)3 +   (0.21)8(0.79)2 
  5   6   7   8 
 30 30  30  29  30   30 
= 1 −   ( 0.05) ( 0.95) +   ( 0.05) ( 0.95) +   ( 0.05) ( 0.95) +   (0.05)3 (0.95)27 
0 1 2 28

 0=0.039 9 1  2  3 
6 X ~ B(15, 0.1)
= 1 − 0.939 2 …
 15 = 0.060 8
a P(X = 3) =   (0.1)3(0.9)12 = 0.129
 3

37
©HarperCollinsPublishers 2018 Cambridge International AS & A Level Mathematics: Probability & Statistics 1 9780008257767

57767_P026_040.indd 37 7/5/18 3:48 PM


3 Discrete random variables

c Y ~ B(20, 0.06077)
Blue beads: X ~ B 10, 5 
 20  14 
P(Y = 5) =   (0.06077)5(0.93923)15
 5  10
( ) (149 ) = 0.074 4 …
6 4
P(X = 6) =   5
= 0.005 02 (3 s. f.)  6  14
d E(X) = 20 × 0.06077 = 1.22 P(exactly six beads of same colour)
= 0.000 96… + 0.007 7… + 0.029 7… + 0.074 4…
 
10 X ∼ B5, 1  P(exactly six beads of same colour) = 0.113(3 s.f.)
 7
14 Let X be the number of bullets that hit the target.
 5
( ) ( 67 ) = 0.021 4 …
3 2
P(3 children) =   1 X ~ B (6, 0.7)
 3 7
P(X  3) = 1–P(X = 0) – P(X = 1) – P(X = 2)
2 3
 5
P(2 children) =    1   6  = 0.128 5 …  6 10  6  9  6 8
= 1 −   ( 0.7 ) ( 0.3) +   ( 0.7 ) ( 0.3) −   ( 0.7 ) ( 0.3) 
0 1 2
 2  7   7 
 0  1  2 
P(3 or 2 children develop the condition)
 6 10  6  9  6 8
= 1 −   ( 0.7 ) ( 0.3) −+   ( 0.7 ) ( 0.3) +   ( 0.7 ) ( 0.3) 
0 1 2
= 0.021 4 … + 0.128 5 … = 0.150
   0  1  2 
11 X ∼ B6, 4 
 9 = 1 – 0.070 47

( ) ( 59 )
 6 4 2 = 0.930
a P(X = 4) =   4 = 0.181 15 a The spins are independent of each other and
 4 9
the order of the outcomes does not matter.
b P(X  4) = P(X = 4) + P(X = 5) + P(X = 6)
b P( R ) = 1 , P( B ) = , P(Y ) =
1 1
 6
( )( )  6
( )( ) ()
 6
4 2 5 1 6 3 6
2
=  4 5 +  4 5 +  4
 4 9 9  5 9 9  6 9
x P(X = x)
= 0.181 + 0.058 + 0.008
= 0.246 1× 1 = 1
2R
2 2 4
12 a X ~ B(10, 0.01)
P(X  1) = 1 − P(X = 0) 1× 1×2 1
1R and 1B 2 3 = 3
 10
=1− (0.01)0(0.99)10
 0 
1× 1×2 1
1R and 1Y 2 6 =6
= 0.095 6
b Y ~ B(5, 0.09562) 1× 1 1
2B 3 3=9
 5
P(Y = 3) = (0.09562)3(0.90438)2 = 0.00715
 3
1× 1×2 1
1B and 1Y =9
( )
1 6 3
13 Red beads: R ~ B 10,
7
1× 1 1
2Y 6 6 = 36
 6  ( 7 ) ( 7 )
 10 1 6 6 4
P(R = 6) = = 0.000 96 …

Yellow beads: Y ~ B(10, )


3 c Let YB be the random spin that landed on
14 yellow from the biased spinner and YF be the

P(Y = 6) =   ( 3 ) ( 11 ) = 0.007 7 …
 10 6 4 random spin that landed on yellow from the
fair spinner.
 6  14 14
P(Y B ) = 1 and P(Y F ) = 1
Green beads: G ~ B(10, 7 )
2 6 3
Therefore, the probability of landing on
 10
( ) ( 75 ) = 0.029 7 …
6 4
P(G = 6) =   2 yellow is: 1 × 1 + 1 × 1 = 1
 6 7 2 6 2 3 4

38
©HarperCollinsPublishers 2018 Cambridge International AS & A Level Mathematics: Probability & Statistics 1 9780008257767

57767_P026_040.indd 38 7/5/18 3:48 PM


3
WORKED SOLUTIONS

 1
d Y ~ Geo 
4
1
19 a The geometric distribution Geo 5 . ()
( ) ( 15 ) = 0.081 9
4 1
Bob has at least three attempts before he b P(X = 5) = 4
5
wins, that is:

( ) = 169
2 4
P(Y  3) = 3  4
c P( X  5) =   = 0.410
4  5
16 The number of weeks from July to October is d Assume that the outcome of each kick is
16, therefore n = 16. The probability of rainfall independent of the results of any other kicks.
is 0.35. Let X be the number of weeks that have
Assume that the probability of scoring a goal
rainfall. This problem can be modelled as binomial
remains constant.
distribution X ~ B(16, 0.35).
P(X > 3) = 1 – P(X  3) 20 a X ~ Geo(0.04)
P(X  3) = 1 − P(X > 3) = 1 − 0.963 = 0.115
 16 16  16  15  16   16 13 
= 1 −   ( 0.35) ( 0.65) +   ( 0.35) ( 0.65) +  b( 0.E(X) 35) (=  )114 + ( 0.35) ( 0.65) 
0 1 2 3
0.65
  = 
25
 0   1  2  0.04   3  
P(X < 25) = 1 − P(X > 24)
 16 16  16  15  16  14  16  
= 1 −   ( 0.35) ( 0.65) +   ( 0.35) ( 0.65) +   ( 0.35) ( 0.65) +   ( 0.35     )3(0.65)13 = 1 − (0.96)24
0 1 2

  0   1   2   3   0.625
     =
6 16  16  15  16  14  16  13  c Y ~ B(10, 0.115)
 ( 0.35) ( 0.65) +  1  ( 0.35) ( 0.65) +  2  ( 0.35) ( 0.65) +  3  ( 0.35) ( 0.65) 
0 1 2 3

 E(Y) = 10 × 0.115 = 1.15


Var(Y) = 10 × 0.115 × (1 − 0.115) = 1.02
 16 13  16  16   16  16 
+   ( 0.35) ( 0.65) +   ( 0.35) ( 0.65) +   ( 0.35) ( 0.65) 
3 34 13 4

 3  34  Assume that whether the machine is stopped on


 4 
each run is independent of the outcomes of the
= 1 – 0.133 86 = 0.866 (3 s.f.) other runs.
Assume that the probability of rainfall each week is or
constant. Assume that the probability that the
Assume that each time it rains is independent of machine is stopped remains constant.
whether it rains in the other weeks.
Mathematics in life and work
17 a There are nine single positive digits, which are
1, 2, 3, 4, 5, 6, 7, 8, 9
X ~ B(100, 0.999)
Therefore, probability of getting a six is P(6) = 1 .  100
9
1 P(X = 99) =  (0.999)99(0.001)1 = 0.0906
 99 
b Let X be the number of digits that the
computer program generated up to and 2 P(X  98) = P(X = 98) + P(X = 99) + P(X = 100)
 1  100
including the first 6. X ~ Geo  = (0.999)98(0.001)2
 9
12  98 
 8 
P(X 13) =   = 0.243
 9  100
+ (0.999)99(0.001)1
1
c P(N=1) = 0.2433 × 9 = 0.0270  99 

 8 1
8  100
d P(N = 9) = 0.2433 ×   × = 0.010 5 + (0.999)100(0.001)0
 9 9  100
18 a X ~ Geo (0.65) = 0.999 8
b P(X = 1) = 0.65 3 P(more than 3 inaccurate bags)
c P(X  5) = 1 – P(X > 5) = 1 – 0.355 = 0.995 = P(X  96) = 1 − P(X  97)
d 200 × 0.65 = 130 people = 1 − [P(X  98) + P (X = 97)]

39
©HarperCollinsPublishers 2018 Cambridge International AS & A Level Mathematics: Probability & Statistics 1 9780008257767

57767_P026_040.indd 39 7/5/18 3:48 PM


3 Discrete random variables

 100
= 1 − 0.999 849 62 −  (0.999)97(0.001)3
 97 
= 1 − 0.999 849 62 − 0.000 146 75
= 3.63 × 10−6
4 P(more than 4 inaccurate bags)
= P(X  95) = 1 − P(X  96)
= 1 − [P(X  98) + P (X = 97) + P (X = 96)]
= 1 − 0.999 849 623 6 − 0.000 146 744 7
 100
− (0.999)96(0.001)4
 96 
= 1 − 0.999 849 623 6 − 0.000 146 744 7
− 0.000 003 562 1
= 6.96 × 10−8

40
©HarperCollinsPublishers 2018 Cambridge International AS & A Level Mathematics: Probability & Statistics 1 9780008257767

57767_P026_040.indd 40 7/5/18 3:48 PM


4
WORKED SOLUTIONS

4 Normal distribution
Please note: Full worked solutions are provided as an aid to learning, and represent one approach to answering
the question. In some cases, alternative methods are shown for contrast.
All sample answers have been written by the authors. Cambridge Assessment International Education bears no
responsibility for the example answers to questions taken from its past question papers, which are contained in this
publication.
Non-exact numerical answers should be given correct to 3 significant figures, or 1 decimal place for angles in
degrees, unless a different level of accuracy is specified in the question.
Where values from the Cambridge Assessment International Education statistical tables are used, the same level of
accuracy has been used in workings unless stated otherwise.
Use graph paper when drawing graphs; graphical solutions given here are not to exact scale but give an indication
of the answer.
Prerequisite knowledge 6 a Random variable A: The red curve
Random variable B: The blue curve
1 a Mean = 3.75 Random variable C: The black curve
b Standard deviation = 2.12
b The mean of random variable B is 4.8.
c New mean = 3.32, new standard deviation = 2.24

2 a E(Y ) = 11
3
σ = 1.31 (3 s.f.)
b P(Y > m + σ ) = P(Y > 4.98) = 1
6
2.4 3.6 4.8 6 7.2 8.4 9.6 10.8
Exercise 4.1A
7 a Always true.
1 a The distribution is symmetrical about the mean. b Never true. The point of inflexion is located one
b The mode, median and mean are all the same. standard deviation on either side of the mean.
c The total area below the curve is 1. c Never true. (It is possible but the probability is 0.)
d The distribution is defined by two parameters: d Always true.
the mean and the standard deviation. 8 a Mean = 2.5 mm
2 Standard deviation is equal. Mean is different for Standard deviation = 0.1 mm
the two distributions.
b 6
 8% of the screws have a diameter measurement
17 within one standard deviation.
3 a 50% b
50
c 47.5% d 47.5% + (99.75 ÷ 2) = 779 9 a 68% of the data is located within one standard
800 deviation.
4 Curve A has a higher mean compared to curve B. 167 ± 1.8 = (165.2, 168.8)
Curve A has a smaller standard deviation
compared to curve B.
5
X
16% 16%

165.2 cm 167.0 cm 168.8 cm

b To find how many standard deviations away


from the mean, 169.7 − 168 = 1.5 standard
1.8
Y deviations, this is about 6.75%  16% − 2.5%  on
 2 
the right of 1σ.
42.5

Standard deviation of blue curve, X, is 1.5 Probability = 0.025 + 0.0675 = 0.0925, so about
Standard deviation of red curve, Y, is 2.5 9.25%. 41
©HarperCollinsPublishers 2018 Cambridge International AS & A Level Mathematics: Probability & Statistics 1 9780008257767

57767_P041_056.indd 41 7/5/18 3:53 PM


4 NORMAL DISTRIBUTION

6 Neither of them were right.


Adam is wrong because both random variable X
and random variable Y have the same variance,
16% 1.52 = 2.25
2.5% Billy is wrong because it is possible to have two
different random variables which have the same
mean and variance.
m s 2s
7 i Graph b and graph c are normal distribution
curves because they show:
10
› symmetry about the centre
› a bell-shaped curve.
ii Graph b – N(21, 1.52)
16% Graph c – N(16.7, 2.32)
8 a Sometimes true. In the case where t = 1, the
m variance 12 is equal to the standard deviation 1
16% of the students failed the exam, which means and so the statement is true. Otherwise, the
that these students did not pass the bench mark. statement is untrue.
From the graph, it shows that this is 1 standard b Sometimes true. Many continuous random
deviation to the left of the mean. variables can be modelled by a normal
Therefore, the pass mark for the test = 68 – 15 = 53 distribution, e.g. heights of people, blood
pressure, errors in measurements, etc.
Exercise 4.2A However, not every continuous random
variable is normally distributed.
1 A ~ N(3, 4), B ~ N(2, 3²)
2 Let A represent Year 1 scores and B represent Year 2 9 a Experiment B
scores. b Experiment B has the shortest average
A ~ N(0, 5), B ~ N(−1, 2²) reaction time; it also has the smallest
3 The ‘N’ and ‘X’ are in incorrect places; the mean variance. This indicates that the reaction
should be 7 and the standard deviation should times are less varied.
be 11.
10 a Let X be the mass of mixed nuts, X ~ N(50, 2.32)
X ~ N(7, 11²)
b
4 a Student A: mean = 5, variance = 49, standard
deviation = 7
Student B: mean = 6, variance = 81, standard
deviation = 9 16% 16%
b
47.7 g 50 g 52.3 g
A
B
Exercise 4.2B

1 a

X a 0 Z

Y
m

42
©HarperCollinsPublishers 2018 Cambridge International AS & A Level Mathematics: Probability & Statistics 1 9780008257767

57767_P041_056.indd 42 7/5/18 3:53 PM


4
WORKED SOLUTIONS

b 5

a 0 Z 0
1.064
c
P(Z > 1.064) = 1 − Φ(1.064)
= 1 − 0.8563
= 0.1437

a 0 b Z 6

–2.872 0

P(Z < −2.872) = Φ(−2.872)


a b 0 Z
= 1 − Φ(2.872)
e = 1 − 0.9979
= 0.0021

0 a Z

2 ‘N’ and ‘Z’ are written in the incorrect positions.


The mean should equal 0. The standard deviation
–1.326 0
should equal 1.
Z ~ N(0, 1²) P(Z > −1.326) = 1 − Φ(−1.326)
3 = 1 − [1 − Φ(1.326)]
= Φ(1.326)
= 0.9076

0 1.423

P(Z < 1.423) = Φ(1.423)


= 0.9226
4

0 1.1 2.1 Z

P(1.1 < Z < 2.1) = Φ(2.1) − Φ(1.1)


= 0.9821 − 0.8643
= 0.1178
0 0.87 Z
P(Z < 0.87) = Φ(0.87)
= 0.8078
43
©HarperCollinsPublishers 2018 Cambridge International AS & A Level Mathematics: Probability & Statistics 1 9780008257767

57767_P041_056.indd 43 7/5/18 3:53 PM


4 NORMAL DISTRIBUTION

9 P(0.24 < Z < 1.102) = Φ(1.102) – Φ(0.24)


= 0.8647 – 0.5948 = 0.2699

Exercise 4.3A

1 a P(0 < Z < 1.2) = Φ(1.2) − Φ(0)


= 0.8849 − 0.5
–1.325 0 1.218 = 0.3849

P(−1.325 < Z < 1.218) = Φ(1.218) − Φ(−1.325) b P(−2.25 < Z < 0) = Φ(0) − Φ(−2.25)

= Φ(1.218) − [1 − Φ(1.325)] = Φ(0) − [1 − Φ(2.25)]

= Φ(1.218) + Φ(1.325) − 1 = Φ(0) + Φ(2.25) − 1

= 0.8884 + 0.9074 − 1 = 0.5 + 0.9878 − 1

= 0.7958 = 0.4878

10 c P(−0.5 < Z < 1.5) = Φ(1.5) − Φ(−0.5)


= Φ(1.5) − [1 − Φ(0.5)]
= Φ(1.5) + Φ(0.5) − 1
= 0.9332 + 0.6915 − 1
= 0.6247

d P(Z > 2) = 1 − Φ(2)


–2.651 –1.43 0
= 1 − 0.9772
P(−2.651 < Z < −1.43) = 0.0228
= Φ(−1.43) − Φ(−2.651) 2 (
a P(X < 35) = P Z <
35 − 30
10 )
= [1− Φ(1.43)] − [1 − Φ(2.651)] = P(Z < 0.5)
= Φ(2.651) − Φ(1.43) = Φ(0.5)
= 0.9960 − 0.9236 = 0.6915
= 0.0724
b P(X > 38.6) = 1 − P(X < 38.6)

( )
11
38.6 − 30
=1−P Z <
10
= 1 − P(Z < 0.86)
= 1 − Φ(0.86)
= 1 − 0.8051
–0.541 = 0.1949

P( Z < – 0.541) = P( Z > 0.541) c P(X > 20) = 1 − P(X < 20)
= 1 − P(Z < 0.541)
= 1 − 0.7057
=1−P Z < ( 20 − 30
10 )
= 1 − P(Z < −1)
= 0.2943
= 1 − Φ(−1)
12 = 1 − [1 − Φ(1)]
= Φ(1)
= 0.8413

d P(35 < X < 40) = P(X < 40) − P(X < 35)

0 1.102 =P Z < (
40 − 30
10
−P Z < ) (
35 − 30
10 )
44
©HarperCollinsPublishers 2018 Cambridge International AS & A Level Mathematics: Probability & Statistics 1 9780008257767

57767_P041_056.indd 44 7/5/18 3:54 PM


4
WORKED SOLUTIONS

= P(Z < 1) − P(Z < 0.5)


 = P( Z > 2.938 )
= Φ(1) − Φ(0.5)
= 1 – Φ (2.938)
= 0.8413 − 0.6915 = 1 – 0.9883
= 0.1498 = 0.0017
 10 − 10.8  = P Z  −0.5 = 1 −  0.5
e P(15 < X < 32) = P(X < 32) − P(X < 15) b P( X  10 ) = P  Z  ( ) ( 5) = 1 − 0.691
 1.6 
=P Z < (
32 − 30
) (
15 − 30
)
− P Z <   10 − 10.8  = P Z  −0.5 = 1 −  0.5
10P( X  10 ) = P  Z10
1.6 
( ) ( 5) = 1 − 0.6915 = 0.3085
= P(Z < 0.2) − P(Z < −1.5) c P(9.2 < X < 11.2) = P( X < 11.2) – P( X < 9.2)
= Φ(0.2) − Φ(−1.5)
= Φ(0.2) − [1− Φ(1.5)]
=P Z < (
11.2 − 10.8
1.6
−P Z <
9.2 − 10.8
1.6 ) ( )
= Φ(0.2) + Φ(1.5) − 1 = P(Z < 0.25) – P( Z < – 1)
= 0.5793 + 0.9332 − 1 = Φ (0.25) – (1 – Φ (1))
= 0.5125 = 0.5987 – (1 – 0.8413)
= 0.44
f P(17 < X < 19) = P(X < 19) − P(X < 17)
5 a X ~ N(6, 1²)
= P  Z < 19 − 30  − P  Z < 17 − 30 
 10   10  P(X < 4.5) = P Z < (4.5 − 6
1 )
= P(Z < −1.1) − P(Z < −1.3) = P(Z < −1.5)
= Φ(−1.1) − Φ(−1.3) = Φ(−1.5)
= [1 − Φ(1.1)] − [1 − Φ(1.3)] = 1 − Φ(1.5)
= Φ(1.3) − Φ(1.1) = 1 − 0.9332
= 0.9032 − 0.8643 = 0.0668
= 0.0389 b P(X > 8) = 1 − P(X < 8)
3 a X ~ N(177, 7²) =1−P Z < 8−6 ( )
( )
1
172 − 177
P(X < 172) = P Z < 7 = 1 − P(Z < 2)
= P(Z < −0.714) = 1 − Φ(2)
= Φ(−0.714) = 1 − 0.9772
= 1 − Φ(0.714) = 0.0228
= 1 − 0.7623
c P(4.5 < X < 8) = P(X < 8) − P(X < 4.5)
= 0.2377
= 0.9772 − 0.0668
b P(X > 180) = 1 − P(X < 180) = 0.9104

=1−P Z <( 180 − 177


) 6 a X ~ N(60, 4²)
( )
7 55 − 60
= 1 − P(Z < 0.429) P(X < 55) = P Z < 4
= 1 − Φ(0.429) = P(Z < −1.25)

= 1 − 0.6660 = Φ(−1.25)

= 0.3340 = 1 − Φ(1.25)
= 1 − 0.8944
c P(172 < X < 180) = P(X < 180) − P(X < 172)
= 0.1056
= 0.6660 − 0.2377
= 0.4283 b P(55 < X < 65) = P(X < 65) − P(X < 55)

4 X ~N(10.8, 1.6 2) (
=P Z <
65 − 60
4 ) (
−P Z <
55 − 60
4 )
 15.5 − 10.8  = P Z > 2.938
a P (X > 15.5) = P  Z >
 1.6  ( ) = P(Z < 1.25) − P(Z < −1.25)
= Φ(1.25) − Φ(−1.25)

45
©HarperCollinsPublishers 2018 Cambridge International AS & A Level Mathematics: Probability & Statistics 1 9780008257767

57767_P041_056.indd 45 7/5/18 3:54 PM


4 Normal distribution

= Φ(1.25) − [1 − Φ(1.25)] = 1 − Φ(1.7)


= Φ(1.25) + Φ(1.25) − 1 = 1 − 0.9554
= 0.8944 + 0.8944 − 1 = 0.0446
= 0.7888
c P(X > 85) = 1 − P(X < 85)
c P(X > 65) = 1 − P(X < 65)
= 1 − P  Z < 85 − 68.5 
=1−P Z < ( 65 − 60
4 ) 
= 1 − P(Z < 3.3)
5 

= 1 − P(Z < 1.25)


= 1 − Φ(3.3)
= 1 − Φ(1.25)
= 1 − 0.9995
= 1 − 0.8944
= 0.0005 = 0.05%
= 0.1056
9 X ~ N(2.025, 0.0152)
d P(65 < X < 68) = P(X < 68) − P(X < 65)
 2.05 − 2.025  = P Z < 1.667
a P( X < 2.05) = P  Z <
0.015 
( )
=P Z <(68 − 60
4
−P Z < ) (
65 − 60
4 ) 
= Φ(1.667 ) = 0.9515 + 0.0007 = 0.9522
= P(Z < 2) − P(Z < 1.25)
= Φ(2) − Φ(1.25) b P( X > 2) = P  Z > 2 − 2.025 
 0.015 
= 0.9772 − 0.8944
= P( Z > −1.667 ) = Φ(1.667 ) = 0.9522
= 0.0828
10 a X ~ N (250, σ 2)
7 X ~ N (168, 4.62)
 158 − 168  = P Z < −2.174 P (X < 235) = 0.005
a P( X < 158 ) = P  Z < ( )
 4.6  Φ(z) = 0.005
=1– Φ (2.174) = 1 – (0.9850 + 0.0002) = 0.0148 z = – 2.576
b P(165 < X < 170 ) = P( X < 170 ) − P( X < 165)     235 − 250 = −2.576
σ
= P  Z < 170 − 168  − P  Z < 165 − 16
68  σ = 5.82
 4.6   4.6 
b P(X > 260) = 1 – P(X < 260) = 1 – Φ(1.717)
= P (Z < 0.435) – P (Z < – 0.652)        = 1 – 0.9570 = 0.0430
= Φ (0.435) – (1 – Φ (0.652))
= (0.664+ 0.0018) – (1 – (0.7422 + 0. 0007))
Exercise 4.3B
= 0.4087 1 a z = Φ−1(0.7642) = 0.720
About 40.9% of the students meet George’s b z = Φ−1(0.638) = 0.353
criteria. c P(Z < z) = 1 − 0.195
8 a X ~ N(68.5, 5²) z = Φ−1(0.805) = 0.859
P(X > 70) = 1 − P(X < 70) d P(Z < z) = 1− 0.375
=1−P Z < ( 70 − 68.5
5 ) z = Φ−1(0.625) = 0.318
e P(Z > z) = P(Z < −z) = 0.8531
= 1 − P(Z < 0.3)
−z = Φ−1(0.8531) = 1.05
= 1 − Φ(0.3)
z = −1.05
= 1 − 0.6179
f P(Z > z) = P(Z < −z) = 0.6
= 0.3281
−z = Φ−1(0.6) = 0.253
b 10% increase of 70 km/h gives 77 km/h z = −0.253
P(X > 77) = 1 − P(X < 77) g P(Z < z) = 0.372 so P(Z < −z) = 1 − 0.372

(
=1−P Z <
77 − 68.5
5 )
−z = Φ−1(0.628) = 0.327
z = −0.327
= 1 − P(Z < 1.7)

46
©HarperCollinsPublishers 2018 Cambridge International AS & A Level Mathematics: Probability & Statistics 1 9780008257767

57767_P041_056.indd 46 7/5/18 3:54 PM


4
WORKED SOLUTIONS

h P(Z < 0) + P(0 < Z < z) = 0.5 + 0.45 b 0.9798 = Φ(z)


0.45 z = 2.05 = x − 30
0.5 10
x = 50.5

c If P(X > x) = 0.1949, P(X < x) = 0.8051


0.8051 = Φ(z)
z = 0.86 = x − 30
0 z 10
x = 38.6
P(Z < z) = 0.95
d 0.75 = Φ(z)
z = Φ−1(0.95) = 1.645
z = 0.674 = x − 30
10
i P(Z < −z) + P(−z < Z < z) = 0.4 + 0.5 x = 36.74
z = Φ−1(0.9) = 1.282 (using the critical value for
the normal distribution table) or 1.281 e P(X < x) = 0.35
P(Z < −z) = 0.65
0.4 0.4
Φ−1(0.65) = −z
z = −0.385 = x − 30
10
0.1 x = 26.15

f If P(X > x) = 0.05, P(X < x) = 0.95


−z 0 z 0.95 = Φ(z)
x − 30
2 Underweight: 0.15 = Φ(z) z = 1.645 =
10
x − 498.7 x = 46.45
z = −1.036 =
7.3
x = 491.14 5 X ~ N(4000, 375²), P(X < x) = 0.9
Therefore, bags of sugar are rejected when the 0.9 = Φ(z)
weight is less than 491.1 g. z = 1.282 = x − 4000
375
Overweight: 0.9 = Φ(z) x = 4480 hours
z = 1.282 = x − 498.7 6 X ~ N(1.753, 0.1²), P(X < x) = 0.929
7.3
x = 508.06 0.929 = Φ(z)
Therefore, bags of sugar are rejected when the z = 1.468 = x − 1.753
0.1
weight is more than 508.1 g. x = 1.90 m
3 a X ~ N(19, 2.4²)
7 X ~ N(68.5, 5²), P(a < X < b) = 0.8
P(X > a) = 0.432, therefore, 0.8 = P(X < b) − P(X > a)
P(X < a) = 1 − 0.432 = 0.568 However, since the normal distribution is
0.568 = Φ(z) symmetrical about the mean,
z = 0.171= a − 19 0.9 = P(X < b), such that a = m − (b − m)
2.4 Therefore, 0.9 = Φ(z)
a = 19.41
z = 1.282 = b − 68.5
b P(X < b) = 0.205 5
b = 74.91, a = 62.09
P(Z > −z) = 0.205 The range of speeds is between 62.1 mph and
1 − 0.205 = Φ(−z) 74.9 mph.
Φ−1(0.795) = −z 8 a Marks scored by students are independent.
z = −0.824 = b − 19 b X ~ N(80, 12²), P(X > x) = 0.25, P(X < x) = 0.75
2.4
b = 17.02 0.75 = Φ(z)
4 a 0.99 = Φ(z) z = 0.674 = x − 80
x − 30 12
z = 2.326 = x = 88.088
10
x = 53.26 Therefore, the lowest mark needed is 89.
47
©HarperCollinsPublishers 2018 Cambridge International AS & A Level Mathematics: Probability & Statistics 1 9780008257767

57767_P041_056.indd 47 7/5/18 3:54 PM


4 Normal distribution

c P(a < X < b) = 0.95, and since the normal b $45 ÷ $1.55 = 29.03 litres
distribution is symmetrical, P(X < b) = 0.975
0.975 = Φ(z)
(
P ( X > 29.03) = P Z > 29.03 − 35
8 )
z = 1.96 = x − 80 = P ( Z > −0.746 ) = Φ ( 0.746 ) = 0.7722
12
c Lorries may also fill up at the petrol station,
b = 103.52, a = 56.48
which will affect the normal distribution
Therefore, the middle 95% of marks will be parameters, as will the amount of fuel
between 56 and 104. remaining in the petrol station.
d P(X > 90) = 1 − P(X < 90) Some people may just visit the counter at the

(
=1−P Z <
90 − 80
12 ) petrol station rather than purchase petrol.
This will also affect the normal distribution
= 1 − P(Z < 0.833) parameters.
= 1 − Φ(0.833) Exercise 4.3C
= 1 − 0.7975
1 X ~ N(43, σ ²)
= 0.2025
P(X > 48) = 0.2
200 × 0.2025 = 40.5 Therefore, P(X < 48) = 0.8
Therefore, 40 students are likely to score 90 0.8 = Φ(z)
or more. z = 0.842 = 48 − 43
s
σ = 5.94
9 X ~ N(190, 2.52)
2 X ~ N( m, 1.3²)
a P (X > x) = 0.2 P(X > 47) = 0.11
1 – Φ(z) = 0.2 Therefore, P(X < 47) = 0.89
Φ(z) = 0.8 0.89 = Φ(z)
47 − µ
z = 0.842 z = 1.227 =
1.3
m = 45.4 cm
     x − 190 = 0.842
2.5 3 X ~ N(m, σ ²)
x = 192.1 g P(X > 332.91) = 0.18, P(X > 325.42) = 0.72
Therefore, P(X < 332.91) = 0.82
b P( –z < Z < z) = 0.4
332.91 − m
Φ(z) – (1 – Φ(z)) = 0.4 = Φ(0.82)
s
Φ(z) = 0.7 332.91 − m
= 0.915
s
z = 0.524 332.91 − m = 0.915σ 1
x − 190 x − 190 Also P(X < 325.42) = 0.28
= 0.524 = −0.524
2.5 2.5 325.42 − m
= Φ(0.28)
x = 191.3, 188.7 s
325.42 − m
Therefore, the range in weights is from 188.7 g to = −0.583
s
191.3 g. 325.42 − m = −0.583σ 2
1 − 2:
c P( X  195) = P  Z  195 − 190  = P( Z  2) 7.49 = 1.498σ
 2.5 
= 1 − Φ( 2) = 1 − 0.9772 = 0.0228
σ = 5 ml
Therefore, m = 325.42 + 0.583(5) = 328 ml
100 × 0.0228 = 2.28
4 X ~ N(m, σ ²)
At least two tins are likely to be 195 g or more.
P(X < 9.92) = 0.25, P(X > 12.24) = 0.25
10 X ~ N(35, 82) P(X < 9.92) = 0.25
 20 − 35  = P Z < −1.875 9.92 − m = Φ(0.25)
a P( X < 20 ) = P  Z < ( )
 8  s
= 1– Φ(1.875) = 1 – (0.9693 + 0.0004) = 0.0303 9.92 − m
= −0.674
s
9.92 − m = −0.674σ 1

48
©HarperCollinsPublishers 2018 Cambridge International AS & A Level Mathematics: Probability & Statistics 1 9780008257767

57767_P041_056.indd 48 7/5/18 3:54 PM


4
WORKED SOLUTIONS

Also P(X < 12.24) = 0.75. = 1 − Φ(1)


12.24 − m = Φ(0.75) = 1 − 0.8413
s
= 0.1587
− m = 0.674
12.24
s c P(87 < X < 95) = P(X < 95) − P(X < 87)
 12.24 − m = 0.674σ 2
= 0.9522 − 0.1587
2 − 1:
2.32 = 1.348σ = 0.7935
σ = 1.721
d X ~ N(μ, σ 2)
Therefore, m = 12.24 − 0.674(1.721) = 11.08
P(X > 85) = 0.1 and P(X < 65) = 0.2
Alternative method to find the mean:
LQ = 9.92 and UQ = 12.24 (
P Z >
85 − µ
σ )
= 0.1
Therefore, µ = 9.92 + 1 (12.24 – 9.92)
2

5

a X ~ N(15.5,
= 9.92 + 1.16 = 11.08
σ 2)
Φ ( σ µ)
85 −
= 0.9

85 − µ

P(X >18) = 0.01 = 1.281
σ
Φ(z) = 0.001
85 – μ = 1.281 σ 1
z = 2.326
18 − 15.5
σ
= 2.326 (
P Z < )
65 − µ
σ
= 0.2

σ = 1.07 cm
b P( X < 14 ) = P  Z < 14 − 15.5  = P( Z < −1.395)

( σ µ)
Φ −
65 −
= 0.8
1.075 
=1 − Φ(1.395) = 1 − (0.9177 + 0.0008) = 0.0815 65 − µ
− = 0.842
σ
8.15% pebbles are smaller than 14 cm.
–(65 – μ) = 0.842 σ  2
6 X ~ N(500, σ 2)
1 + 2
a P(X < 480) = 0.1
20 = 2.123 σ
1 – Φ(z) = 0.90
σ = 9.42 minutes
– z = – 1.282
Therefore, μ = 85 – 1.281 × 9.42 = 72.9 minutes
480 − 500
= −1.282
σ
8 X ~ N( m, σ ²)
σ = 15.6 g P(X ⩾ 140) = 0.2, P(X ⩾ 130) = 0.8

b P(X > 508) = P Z > ( 508 − 500


15.60 )
= P ( Z > 0.506 )
Therefore, P(X < 140) = 0.8
140 − m = Φ−1(0.8)
= 1 – Φ(0.506) s
140 − m = 0.842
= 1 – (0.6915 + 0.002) = 0.3065 s
7 a X ~ N(90, 3²) 140 − m = 0.842 σ 1
P(X > 95) = 1 − P(X < 95) Also P(X > 130) = 0.8
=1−P Z < (
95 − 90
3 ) P(Z > z) = 0.8
P(Z < −z) = 0.8
= 1 − Φ(1.667) 130 − m
= −Φ−1(0.8)
= 1 − 0.9522 s
= 0.0478 130 − m
= −0.842
s
b P(X < 87) = P Z < ( 87 − 90
3 ) 1 − 2:
130 − m = −0.842σ 2

= P(Z < −1) 10 = 1.684σ


= Φ(−1) σ = 5.94 kg
Therefore, m = 140 − 0.842(5.94) = 135 kg   Kk

49
©HarperCollinsPublishers 2018 Cambridge International AS & A Level Mathematics: Probability & Statistics 1 9780008257767

57767_P041_056.indd 49 7/5/18 3:54 PM


4 Normal distribution

9 X ~ N(12, 22) Exercise 4.4A

a P( X < 11) = P  Z < 11 − 12  1 a m = np = 20 × 0.5 = 10


 2 
b Variance: σ 2 = np(1 − p) = 20 × 0.5 × 0.5 = 5
= P( Z < −0.5) = 1 − Φ( 0.5)
Standard deviation: σ = 5 = 2.236
= 1 − 0.6915 = 0.3085

b P ( X > 13.5) = P Z > (


13.5 − 12
2 ) c P(15.5 < X < 16.5) = P (15.52.236− 10 < Z < 16.52.236− 10 )
= P(2.460 < Z < 2.907)
= P ( Z > 0.75) = 1 − Φ ( 0.75)
= Φ(2.907) – Φ(2.460)
= 1 − 0.7734 = 0.2266
= 0.9982 − 0.9931
c P( X < 10 ) + P( X >P13 ( X −>1213) = P Z < 10 −2 12 
( X) =<P10 )Z+<P10 = 0.0051
 2 
12 10−13 12− 12 
P( X < 10 ) +( P( X > 13
< ) ) =+( PP13
+ > −)12  
= − <
 2 2= P( Z+P<2−1) 2+ P(Z=>P0(.Z 5) < −1) + P( Z > 0.5) 2 a µ = 108 × 0.88 = 95.04
P X 10 P X 13
Z< 10 P Z 

 13 − 12  = P Z < −1 + P Z > 0.5 σ2 = 108 × 0.88 × 0.12 = 11.4048


 − +  = P Z <−1 +( P Z > 0 ) .5) ( )
 2   (2  ) (
+P  13 12 P σ = 3.377
= (1 – Φ (1)) + (1 – Φ (0.5)) = (1 – 0.8413) + 0.3085 (
P(X < 99.5) = P Z <
99.5 − 95.04
3.377 )
= 0.4672         = Φ(1.321)
d P(X > 13) = 0.05    and   P(X < 10) = 0.12       = 0.9067
1 – Φ(z1) = 0.95   and   1 – Φ(–z2) = 0.88 b P(85.5 < X < 104.5)

13 − µ
z1 = 1.645        –z2 = 0.1171
(
= P 85.5 − 95.04 < Z < 104.5 − 95.04
3.377 3.377 )
= 1.645 = P(Z < 2.801) − P(Z < −2.825)
σ
13 – µ = 1.645σ 1 = Φ(2.801) – (1 − Φ(2.825))
10 − µ = 0.9975 − (1 − 0.9976)
   σ = −1.171
= 0.9951
–10 + µ = 1.171σ 2
1 − 2: 3 m = np = 200 × 0.32 = 64
3 = 2.816σ Variance: σ 2 = np(1 − p) = 200 × 0.32 × 0.68 = 43.52

σ = 1.07 minutes Standard deviation: σ = 43.52 = 6.597

Therefore, µ = 13 – 1.645 × 1.07 = 11.2 minutes (


129.5 − 64
a P(X < 129.5) = P Z < 6.597 )
= Φ(9.93)
)
4.52)

) (
10 X ~ N(16,
12 − 16
( P ( X < 12) + P12
a P ( X < 12) + P ( X > 18 ) = P Z <
4.5
18 ) = P Z <
( X −>16 4.5 ≈1

)() ( ) )
P  Z > 50.5 − 64  = P(Z > − 2.046)
((
12 − 16
− b P(X > 50.5) =
( X < 12) P+(PX( X< 12 ) +)P=( PX 18
> 18 >Z 18
< )12= −P16Z <+ P4.5
− 16
18 16
= P ( Z < −0.889) + P ( Z > 0.444 )  6.597 
+P 4.5 4.5 ) + P ( Z > 0.444 )
= P ( Z < −0.889
4.5
P ( )( )
18 − 16+ P 18 − 16 = P ( Z < −0.889) + P ( Z > 0.444 )
4.5
Z < −0.889) + P ( Z > 0.444 )
= P (4.5
= P( Z <
= Φ(2.046)
2.046)

= (1 – Φ (0.889)) + (1 – Φ (0.444))
= 0.9796
( )
= (1 – (0.8106 + 0.0025)) + (1 – (0.67 + 0.0014)) 75.5 − 64
c P(X > 75) = P Z >
= 0.5155 6.597
b P(X < x) = 0.9, Φ(z) = 0.9, z = 1.282 = 1 − Φ(1.743)
x − 16 = 1 − 0.9594
= 1.282
4.5
= 0.0406
x = 21.769, so 22 minutes
d P(75 < X < 130) = P(X < 129.5) − P(X < 75.5)
c X ~ N(16, σ 2)
= 1 − 0.9594
P(X < 19) = 0.99, Φ(z) = 0.99, z = 2.326
19 − 16 = 0.0406
= 2.326 4 a 
You may use the normal distribution as an
σ
σ = 1.29 minutes approximation for the binomial B(n, p) (where n
is the number of trials each having probability p
of success) when:
50
©HarperCollinsPublishers 2018 Cambridge International AS & A Level Mathematics: Probability & Statistics 1 9780008257767

57767_P041_056.indd 50 7/5/18 3:54 PM


4
WORKED SOLUTIONS

n is large b P ( 44.5  X  45.5) = P ( X  45.5) − P ( X  44.5)


np > 5 and n(1 – p) > 5
This ensures that the distribution is
( 5.657 ) (
= P Z  45.5 − 40 − P Z  44.5 − 40
5.657 )
reasonably symmetrical and not skewed at = P(Z , 0.972) – P(Z , 0.795)
either end.
= Φ(0.972) – Φ(0.795)
b P(T < 5) = 0.2173
= (0.8340 + 0.0005) – (0.7852 + 0.0015) = 0.0478
c Mean = m = 15 × 0.4 = 6
8 a X ~ B (30, 0.12)
Variance = σ 2 = np(1 − p) = 15 × 0.4 × 0.6 = 3.6
P(X < 3) = P(X = 0) + P(X = 1) + P(X = 2)
Standard deviation = np(1 − p) = 1.897
 30  30
=   ( 0.12) ( 0.88 ) +   ( 0.12) ( 0.88 )
0 30 1 29

(
P(T < 5) = P Z <
4.5 − 6
1.897 ) 0
 30
=   30(0.12
 )0 (0.88
1 
 30
)30 +28  (0.12)1 (0.88 )29
 0+  1 
 2  ( 0.12) ( 0.88 )
2
= P(Z < −0.791)
= 1 − Φ(0.791)  30
+   ( 0.12) ( 0.88 )
2 28

= 1 − 0.7855 2
= 0.2145 = 0.2847

d
(0.2173 − 0.2145) × 100% = 1.29%
b µ = np = 60 × 0.12 = 7.2
0.2173 np (1 – p) = 60 × 0.12 × 0.88 = 6.336
5 n = 120, p = 0.5 σ = 6.336 = 2.517
µ = np = 120 × 0.5 = 60
np(1 – p) = 120 × 0.5 × 0.5 = 30
(
P ( X < 8.5) = P Z <
8.5 − 7.2
2.517 )
= P ( Z < 0.516 )

= Φ ( 0.516 ) = 0.6971
σ = 30 = 5.477

( )
54.5 − 60 9 a To find the probability of at least 13:
P ( X  54.5) = P Z  = P ( Z  −1.004 )
5.477 15C
15 ×
15 0 15 14
 0.72 × 0.28 + C14 × 0.72 × 0.281
15 13
+ C13 × 0.72 × 0.28 2
= Φ(1.004) = 0.8413 + 0.0009 = 0.8422
So: P(at least 13) = 0.1645
6 a X ~ B(50, 0.35)
 50  b Mean = m = 15 × 0.72 = 10.8
P ( X = 20 ) =   ( 0.35) ( 0.65) = 0.0875
20 30

 20 Variance = σ 2 = np(1 − p) = 15 × 0.72 × 0.28 = 3.024


b µ = np = 50 × 0.35 = 17.5 Standard deviation = np(1 − p) = 1.739
np (1 – p) = 50 × 0.35 × 0.65 = 11.375 P(X  13) = 1 − P(X < 12.5)

( )
σ = 11.375 = 3.373 12.5 − 10.8
=1−P Z <
X ~ N(17.5, 11.375) P (19.5  X  20.5) 1.739
=(19.5
X
X ~ N(17.5, 11.375) P ~ (N(17.5,
P XX 
20.5 − P ()XP (
)20.5
11.375) 19.5 ) X  20.5)
19.5 = 1 − Φ(0.978)
= P ( X  20.5) − P ( X=
=P Z(
P (19.5 20.5) −−17.5
X ) 20.5
) (
P ( X  19.5) 19.5 − 17.5
−P Z  ) = 1 − 0.8360

( ) ((
= P Z  20.5 − 17.5 =−PPZZ 19.5
3.373
3.373
20.5 −
3.373 )) ( 3.373
− 17.5 − P Z  19.5
17.5
3.373
− 17.5
3.373 ) = 0.1640

= P(Z  0.889) – P (Z  0.593) c (0.1645 − 0.1640) × 100% = 0.3% error, so the two
0.1645
= Φ(0.889) – Φ(0.593)
values agree closely.
= (0.8106 + 0.0025) – (0.7224 + 0.0010) = 0.0897
d The agreement would be worse, as the normal
7 µ = np = 200 × 0.2 = 40 distribution is a better approximation when
np (1 – p) = 200 × 0.2 × 0.8 = 32 p is closer to 0.5. When p = 0.85, n(1 − p) = 2.25,
which is smaller and further from 5 than its
σ = 32 = 5.657 value when p = 0.72 (4.2).
Approximate X ~ B(200, 0.2) with X ~ N(40, 32) 1
10 a P(any one correct) =
5
(
28.5 − 40
a P ( X  29) = P Z > 5.657 = P ( Z > −2.033)) b Mean = m = 35 × 0.2 = 7
Variance = σ 2 = np(1 − p) = 35 × 0.2 × 0.8 = 5.6
= Φ ( 2.033) = 0.9790

51
©HarperCollinsPublishers 2018 Cambridge International AS & A Level Mathematics: Probability & Statistics 1 9780008257767

57767_P041_056.indd 51 7/5/18 3:54 PM


4 Normal distribution

c P(X < a) = 0.99 = 1 − Φ (1.333)


Φ−1(0.99) = z = 0.0913
z = 2.326
0.0913 × 12 = 1.09 months
a−7
2.326 =
5.6 b P(Y < 53) = P Z < 53 − µ  = 0.38
a – 7 = 2.326 × 5.6  5 
a = 12.504 53 − µ
Φ−1(0.38) =
So the pass mark should be 13. 5
53 − µ
Exam-style questions −0.305 =
5
−1.525 = 53 − m
1 a X ~ N(1005, 2²)

( )
     m = 54.5 mm
1000 − 1005
P(X < 1000) = P Z <
= P(Z < −2.5)
2
(
c P(Y < 60) = P Z <
60 − 54.53
5 )
= Φ(−2.5) = Φ(1.094)
= 1 − Φ(2.5) = 0.863
= 1 − 0.9938  12
P(exactly 4 months) =   (0.863)4(0.137)8
= 0.0062  4
b P(X > 1007) = 1 − P(X < 1007) = 0. 000 034 1

=1−P Z < (1007 − 1005


) 5 a X ~ N(1000, 110²)

( )
2
900 − 1000
P(X > 900) = P Z >
= 1 − Φ(1) 110
= 1 − 0.8413 = P(Z > −0.909)
= 0.1587 = Φ(0.909)
= 0.8182
c P(1000 < X < 1007) = 1 − (0.1587 + 0.0062)
= 0.8351 (
b P(X > 1200 hours) = P Z > 1200 − 1000
110 )
So the probability that they both contain = P(Z > 1.818)
between 1000 g and 1007 g is 0.83512 = 0.697. = 1 − Φ(1.818)
2 (
a P(taller than 160 cm) = P Z > 160 − 164.5
8.75 ) = 0.0345
= Φ(0.514) c P(900 < X < 1000) = P(X < 1000) − P(X < 900)
= 0.6964 = P Z <(1000 − 1000
110 )
( )
b P(H < h) = 0.45 900 − 1000
−P X <
P(Z < z) = 0.45 110
P(Z < −z) = 0.55 = P(Z < 0) − P(Z < −0.909)
Φ−1(0.55) = −z = Φ(0) − Φ(−0.909)
z = −0.125 = h − 164.5 = Φ(0) − [1 − Φ(0.909)]
8.75
h = 163.4 cm = 0.5 − 1 + 0.8182

3 P(X > 320) = 1 – P(X < 320) = 0.3182


 320 − 4( 72) 
=1−Φ   d P(X < x) = 0.01
 (
 4 × 10 2 
 ) P(Z < z) = 0.01
= 1 − 0.9452 P(Z < −z) = 0.99
= 0.0548 −z = 2.326
4 a X ~ N(82, 62) x − 1000
= −2.326
110
(
P(X > 90) = P Z >
90 − 82
6 ) x = 744.14 hours
or 31 days to the nearest day.
52
©HarperCollinsPublishers 2018 Cambridge International AS & A Level Mathematics: Probability & Statistics 1 9780008257767

57767_P041_056.indd 52 7/5/18 3:54 PM


4
WORKED SOLUTIONS

6 a X ~ B(30, 0.2) b X ~ N(m, 0.13²)


Approximated by Y ~ N(6, 4.8) since P(X < 3.54) = 0.04
np = 6 > 5 and nq = 24 > 5. 3.54 − µ 
= P Z <
b P( X  4) = P(Y > 3.5)  0.13 

 3.5 − 6  z = −Φ−1(0.96)
= P Z > 
 4.8   3.54 − µ  = −1.751
 0.13 
= P( Z > −1.141) 3.54 − m = −1.751× 0.13
= Φ(1.141) = 0.8731 m = 3.77 litres
c P(X  4 on each of two days)
c X ~ N(3.58, σ ²)
= 0.8731 × 0.8731 = 0.7623 P(X < 3.54) = 0.04
7 a X ~ N(m, σ ²)
P(X > 212.6) = 0.33
( 3.54 − 3.58
=P Z <
s )
z = −Φ−1(0.96)
Therefore, 0.33 = 1 − P(X < 212.6) 0.04 = −1.751

= 1 − P  Z < 212.6 − µ  s
 σ  σ = 0.0228 litres
Φ−1(0.67) =
212.6 − µ 9 X ~ N(416, 8σ ²)
σ
P(X > 400) = 0.99
212.6 − µ
0.440 =
212.6 − m = 0.44σ
σ
1
(
Therefore, P Z > 400 − 416 = 0.99
2.828s )
Also 0.12 = P(X < 211.8)
P(Z < z) = 0.12 is the same as P(Z < −z) = 0.88.
P Z< ( −16
2.828s ) = 0.01
−16 = −2.326
Φ−1(0.88) = −  211.8 − µ  2.828s
 σ 
σ = 2.432
−211.8 + m = 1.175σ 2
1 + 2: So the standard deviation is 2.43 g.
0.8 = 1.615σ
10 a The number of trials is large, making the
σ = 0.495
binomial distribution impractical as a model.
Therefore, m = 212.6 – 0.44(0.495) = 212.38
b np = 2500 × 0.38 = 950 and nq = 2500 × 0.62 = 1550
P(X > 212) = P  Z > 212 − 212.38 
 0.4954  Since np  5 and nq  5, a normal
  = P(Z > −0.767) approximation can be used.
  = P(Z < 0.767) c m = np = 950 and σ 2 = npq = 589, so X ~ N(950, 589)
  = 0.7785 d 40% of the vote is 1000 votes.
b P(accepted) = P(211.8 < X < 212.8)  
= P(X < 212.8) − P(X < 211.8) P( X > 1000 ) = P  Z > 1000 − 950 
 589 
= P  Z < 212.8 − 212.38  − 0.12
 0.4954  = P(Z > 2.06)
= 0.8017 − 0.12 = 1 − P(Z < 2.06)
= 0.682
= 1 − 0.9803
Therefore, 31.8% are rejected because they are
outside this range. = 0.0197

8 a X ~ N(3.58, 0.13²) 11 X ~ N(75, 122)

(
P(X < 3.54) = P Z <
3.54 − 3.58
0.13 ) a From the tables: P(Z < 1.281) = 0.9
Therefore, 1.281 =
x − 75
12
⇒ x = 90.372
= P(Z < −0.308)
Assuming that only whole marks are
= Φ(−0.308) awarded, students must score at least 91
= 1 − Φ(0.308) marks to be in the top 10%.
= 1 − 0.6209
= 0.3791
53
©HarperCollinsPublishers 2018 Cambridge International AS & A Level Mathematics: Probability & Statistics 1 9780008257767

57767_P041_056.indd 53 7/5/18 3:54 PM


4 Normal distribution

b From the tables: P(z < 0.674) = 0.75 From the tables: P(z < 1.724) = 0.9577
Therefore, 0.674 = x − 75 ⇒ x = 83.1 Therefore, 1.724 = 15 − 8.75 ⇒ σ = 3.63 minutes
12 s
By symmetry, the middle 50% of the
students score between 67 marks and 83 b P(X < 5) = P z < 5 − 8.75 
marks (to nearest mark).  3.625 

( )
= P(z < −1.034)
c P(X < 40) = P Z < 40 − 75
12 = 1 − P(z < 1.034)

= P(Z < −2.917) = 1 − 0.8494

= 1 − P(Z < 2.917) = 0.1506

= 1 − 0.9982 c 15 000 × 0.1506 = 2259 patients

= 0.0018 14 M ~ B(200, 0.44)

12 X ~ N(55, 172) np = 200 × 0.44 = 88

a Equal proportions means equal probabilities nq = 200 × 0.56 = 112

( 42 − 55
P(X < 42) = P Z < 17 ) Since 88 > 5 and 112 > 5, a normal approximation
can be applied.
= P(Z < −0.765) np(1 – p) = 200 × 0.44 × 0.56 = 49.28
= 1 − P(Z < 0.765) M ~ N (88, 49.28)
= 1 − 0.7779
P ( 75.5 < M < 99.5) = P ( M  99.5) − P ( M  75.5)
= 0.2221
   
The 0.7779 remaining for standard eggs and = P  Z  99.5 − 88  − P  Z  75.5 − 88 
 49.28   49.28 
large eggs needs to be divided equally. This

( )
means that P(X < x) = 1 − 0.7779 = 0.6111
2
= P(Z , 1.638) – P (Z , – 1.781)
= Φ (1.638) – (1 – Φ (1.781))
From the tables: P(z < 0.282) = 0.6111 = (0.9484 + 0.0008) – 1 + (0.9625 + 0.0001) = 0.912
Therefore, 0.282 = x − 55 ⇒ x = 59.79
17 15 a X ~ B(12 000, 0.003)
So eggs weighing more than 59.8 grams
b μ = np = 12 000 × 0.003 = 36
should be considered to be large eggs.
σ 2 = np (1 – p) = 12000 × 0.003 × 0.997 = 35.892
b X ~ B(1000, 0.01)
c np = 36
np = 1000 × 0.01 = 10 and nq = 1000 × 0.99 = 990.
nq = 12 000 × 0.997 = 11 964
Since np ≥ 5 and nq ≥ 5, a normal
Since 36 > 5 and 11964 > 5, a normal
approximation can be used.
approximation can be applied.
X ~ N(10, 9.9)
Y ~ N (36, 35.892)
To calculate P(X = 10), we need to use a
P(29.5  Y  30.5) = P(Y  30.5) − P(Y  29.5)
continuity correction.
 30.5 − 36  P  Z 29.5 − 36 
P(X = 10) ≈ P(9.5 < X < 10.5) = P Z   −   
 35.892  35.892 
≈ P(X < 10.5) − P(X < 9.5)
= P(Z  −0.918) − P(Z  −1.085)
 10.5 − 10   9.5 − 10  = (1 − Φ(0.918)) − (1 − Φ(1.085))
≈ PZ <  − P  Z < 
 9.9  9.9 
= Φ(1.085) − Φ(0.918)
≈ P(Z < 0.159) − P(Z < −0.159)
= (0.8599 + 0.0012) − (0.8186 + 0.002) = 0.0405
≈ P(Z < 0.159) − [1 − P(Z < 0.159)]
16 T ~ N(75, 152)
( )
≈ 2P(Z < 0.159) − 1
60 − 75
a i P (T < 60 ) = P Z < = P ( Z < −1)
≈ 2 × 0.5632 − 1 15
= 0.1264 = 1 − Φ (1) = 1 − 0.8413 = 0.1587
13 P(X > 15) = 0.0423 and X ~ N(8.75, σ 2) ii P ( 60  T  90 ) = P (T  90 ) − P (T  60 )
a P(X < 15) = 0.9577
(
=P Z <
90 − 75
15 ) (
−P Z <
60 − 75
15 )
54
©HarperCollinsPublishers 2018 Cambridge International AS & A Level Mathematics: Probability & Statistics 1 9780008257767

57767_P041_056.indd 54 7/5/18 3:54 PM


4
WORKED SOLUTIONS

 = P(Z < 1) – P(Z < – 1) = Φ(1) – (1 – Φ(1)) 19 W ~ B (12, 0.48)


 = 0.8413 – 1 + 0.8413 = 0.6826  12
a P ( X = 4 ) =   ( 0.48 ) ( 0.52) = 0.1405
4 8

b Since the mean time a user spends in the  4


library is 75 minutes, which is more than an
hour, a user coming after 9.00 pm can only b 3 of a bag = 9 bulbs
4
spend under the mean time.
P(X > 9) = P(X = 10) + P(X = 11) + P(X = 12)
c You know that 99.75% the data lies within ±3 σ.
 12  2  12 
 ( 0.48 ) ( 0.52) +  11  ( 0.48 ) ( 0.52)
10 11 1
m = 75   σ = 15 =
 10   
m + 3σ = 75 + 3 × 15 = 120 minutes = 2 hours
 12 
 ( 0.48 ) = 0.0137
12
10 – 2 = 8 +
 12 
The latest time of entry to the library is
about 8 pm. c n = 50, p = 0.48
17 a n = 300, p = 0.02 np = 50 × 0.48 = 24
np = 300 × 0.02 = 6 nq = 50 × 0.52 = 26
nq = 300 × 0.98 = 294 Since 24 > 5 and 26 > 5, a normal approximation
Since 6 > 5 and 294 > 5, a normal approximation can be applied.
can be applied. np(1 – p) = 50 × 0.48 × 0.52 = 12.48
np (1 – p) = 300 × 0.04 × 0.98 = 5.88 X ~ N (24, 12.48)
X ~ N (6, 5.88)
 
  P( X > 30.5) = P  Z > 30.5 − 24 
P( X < 4.5) = P  Z < 4.5 − 6   12.48 
 5.88 
= P( Z > 1.840 ) = 1 − Φ(1.840 )
= P( Z < −0.619) = 1 − Φ( 0.619)
= 1 – 0.9671 = 0.0329
= 1 – (0.7291 + 0.0029) = 0.268
20 X ~ B (n, 0.07)
b The cost to produce 300 dolls: 1.50 × 300 = $450
The expected number of damaged dolls a E(X) = 8
= 300 × 0.02 = 6 np = n × 0.07 = 8    n = 114.25   
The revenue from the dolls = (300 – 6) × 10 = $2940 Therefore, 115 people
Therefore, the expected profit = $2940 – $450 b np = n × 0.07 = 2    n = 29
= $2490 σ 2 = np(1 – p) = 29 × 0.07 × 0.93 = 1.8879
18 a W ~ N (500, 152) Therefore, σ = 1.8879 = 1.374
P(W < w) = 0.1
Mathematics in life and work
Φ (–z) = 0.9
z = − 1.282 1 P(X > 308 g) = 1 − P(X < 308)
w − 500 = −1.282
15
=1−Φ (
308 − 300
11 )
w = 480.8 = 1 − 0.7663
= 0.2337
b P ( 505  W  515) = P (W  515) − P (W  505)

( ) ( )
515 − 500 505 − 500 2 Find m such that P(X < m) = 0.02
=P Z < −P Z < P(Z < z) = 0.02
15 15
P(Z < −z) = 0.98
= P(Z < 1) – P(Z < 0.333) = Φ(1) – Φ(0.333)
−z = Φ−1(0.98)
= 0.8413 – (0.6293 + 0.0011) = 0.2109
z = −2.054
Let Y be the number of Chinese leaf cabbages
m − 300 = −2.054
that Amy picked, Y ~ B (5, 0.211) 11
 5 2  5 1  5 m = 277.4 = 277 g (3 s.f.)
P (Y  3) =   ( 0.211) ( 0.789) +   ( 0.211) ( 0.789) +   ( 0.211) = 0.0667
3 4 5
 3  4 3 5Y ~ N(m, σ ²),
P(Y > 290) = 0.95, P(Y < 305) = 0.97
 5 2  5 1  5 290 − m
P (Y  3) =   ( 0.211) ( 0.789) +   ( 0.211) ( 0.789) +   ( 0.211) = 0.0667
3 4 5
Therefore, Φ−1(0.95) = −1.645 =
 3  4  5 s

55
©HarperCollinsPublishers 2018 Cambridge International AS & A Level Mathematics: Probability & Statistics 1 9780008257767

57767_P041_056.indd 55 7/5/18 3:54 PM


4 Normal distribution

290 − m
−1.645 =
s
290 − m = −1.645 σ 1
305 − m
Also, Φ−1 (0.97) = 1.881 =
s
305 − m
1.881 =
s
305 − m = 1.881σ 2
2 − 1:
15 = 3.526 σ
σ = 4.25 g
Therefore, m = 305 − 1.881(4.25) = 297 g

56
©HarperCollinsPublishers 2018 Cambridge International AS & A Level Mathematics: Probability & Statistics 1 9780008257767

57767_P041_056.indd 56 7/5/18 3:54 PM


WORKED SOLUTIONS

Summary Review
Please note: Full worked solutions are provided as an aid to learning, and represent one approach to answering the
question. In some cases, alternative methods are shown for contrast.
All sample answers have been written by the authors. Cambridge Assessment International Education bears no
responsibility for the example answers to questions taken from its past question papers, which are contained in this
publication.
Non-exact numerical answers should be given correct to 3 significant figures, or 1 decimal place for angles in
degrees, unless a different level of accuracy is specified in the question.
Use graph paper when drawing graphs; graphical solutions given here are not to exact scale but give an indication
of the answer.

Warm-up Questions iii Drawing a line from 2200 hours shows:

1 i 0.15 + 0.22 + 0.18 + 0.24 = 0.79 200 ×

The sum of the probabilities must sum to 1, ×


150

Cumulative
so missing value is 1 – 0.79 = 0.21.

frequency
ii P(lemon or lime) = 0.15 + 0.22 = 0.37 100 ×
9
2 Paul: = 0.257…
35 50
×

Sammy: 26% = 0.26 ×


T
0 500 1000 1500 2000 2500 3000 3500
Therefore, Sammy is more likely to win because he Lifetime (hours)
has the higher probability.

3 a µ≈ ∑ fx = [(500 × 10) + (1250 × 30) + (1750 × 55) Therefore, the number of bulbs lasting more
∑ f + (2250 × 72) + (3000 × 33)] ÷ 200 than 2200 hours is 200 – 120 = 80 bulbs.
NB – this estimate could be as low as
µ ≈ 399 750 68 depending on the quality of the diagram.
200
μ ≈ 1999 hours c P(exactly one bulb > 3500 hours) = (109 × 52 ) + (101 × 35 )
b i Lifetime, T hours
T  1000
Number of bulbs
P(exactly one bulb > 3500 hours) =
10
(9 2
×
10 5
+
1 3
×
10 5 )( )
18 3 21
T  1500 40 = + =
50 50 50
T  2000 95
167
A Level Questions
T  2500
T  3500 200 1 X ~ N(μ, 0.7142) and the area we are interested
in is:
ii Be sure to plot cumulative frequency
against the upper class boundaries. 0.475

200 ×

×
150
Cumulative
frequency

100 ×
3.2 μ
50
×
Using the cumulative normal distribution tables
×
0
× T in reverse, we can say that P(Z < 1.96) = 0.975.
500 1000 1500 2000 2500 3000 3500
Lifetime (hours) Therefore:

57

57767_P057_066.indd 57 7/5/18 4:09 PM


Summary REVIEW

3.2 − µ 4 i For type A, there are 61 smartphones, so the


−1.96 =
      0.714 median is the 31st item in the ordered list.
−1.3994 … = 3.2 − μ The median is 0.52 seconds.
μ = 4.5994 … = 4.60 m (3 s.f.) The lower quartile is between the 15th and
2 X ~ B(11, 0.76) 16th items in the ordered list.
The lower quartile is:
P(X < 10) = 1 − [P(X = 10) + P(X = 11)]
0.40 + 0.42 = 0.41 seconds
 11  11  2
= 1 −   × 0.7610 × 0.24 +   × 0.7611  The upper quartile is between the 15th and
 10  11  16th items from the end of the ordered list.
 11  11  The upper quartile is:
= 1 −   × 0.7610 × 0.24 +   × 0.7611 
  10   11  0.78 + 0.80 = 0.79 seconds
2
= 0.781
ii
3 i P(1st die divisible by 3 and 2nd die not divisible Type A
2 4 2
by 3) = × =
6 6 9
But to find P(A), we need to consider that Type B
this could happen the other way around, i.e.
with the 1st die not divisible by 3 and the
2nd die divisible by 3. 0 0.2 0.4 0.6 0.8 1
Time (s)
Therefore, P( A) = 2 × 2 = 4
9 9
iii T
 he median for B is lower than for A, so
We know that the product of two smartphone B is quicker because it takes
odd numbers is an odd number. less time on average to load the apps. The
So the probability of an odd product is: interquartile range for B is narrower too, so
3 3 9 1 smartphone B is more consistent.
× = =
6 6 36 4
1 3 5 a i If we treat the 4 Es as though they are one
Therefore, P(B) = 1 − =
4 4 block, then we get (EEEE)***** and there are
4 3 1
So P(A) × P(B) = × = 6! arrangements of this list. However, there
9 4 3
To find P(A ∩ B) it is useful to draw a sample are 2 Ns and 2 Ss, so we need to divide by 2!
space diagram. twice to remove the duplicate arrangements.
The number of arrangements is 6! = 180 .
1 2 3 4 5 6 2!2!
1 ii Suppose that S is fixed in place at the
beginning of the list and that T is fixed
2
in place at the end of the list: S*******T.
3 There are 7! arrangements, but there
4 are 2 Ns and 4 Es, so we need to divide
5 by 4!2!. We get 7! = 105 . But we
4!2!
6 could repeat the process with S and T
represents event A reversed, which doubles the number of
represents event B arrangements to 210.
Yellow highlighting represents A ∩ B
b There are 3 Es available. Assume that one
of them is selected and that the others are
Therefore, P(A ∩ B) = 12 = 1 . then unavailable. We then need to choose 3
36 3
letters from the remaining 6 letters, which is
Because P(A ∩ B) = P(A) × P(B), we can 6C = 20.
3
conclude that A and B are independent.
There are 20 possible selections that contain
ii Since P(A ∩ B) > 0, we can say that A and B
exactly one E.
are not mutually exclusive.

58

57767_P057_066.indd 58 7/5/18 4:09 PM


WORKED Solutions

6 i 1st 2nd 3rd iii The lower quartile is approximately the 28th
Attempt Attempt Attempt person, so in class 6–20.
The upper quartile is approximately the 83rd
S person, so in class 61–80.
0.4
IQRmin = UQmin − LQmax
U S IQRmin = 61 − 20 = 41
0.6 0.4
8 µ = 5 − 2 + 12 + 7 − 3 + 2 − 6 + 4 + 0 + 8 = 2.7
S 10
U
S = P (successful) 0.6 0.4 Var(X) = E(X2) − E2(X)
U = P (unsuccessful)
U Var( X ) = [52 + (−2)2 + 122 + 7 2 + (−3)2 + 22 + (−6)2 + 4 2 + 0 2 + 8 2] ÷ 10
Var( X ) = [52 + (−2)2 + 0.6
122 + 7 2 + (−3)2 + 2
2
+ (−6)2     + 4 2 + 0 2 + 8 2] ÷ 10 − 2.7 2
ii P(X = 0) = 0.4 Var(X) = 35.1 − 2.72 = 27.8
P(X = 2) = 0.62 × 0.4 = 0.144 9 i The upper quartile is in the class ‘5.5–7.0’ cm.
P(X = 3) = 0.63 = 0.216 ii Firstly, find the class widths, then each
So the completed table is: frequency density as shown in the table below.
x 0 1 2 3 Length
2.0–3.5 3.5–4.5 4.5–5.5 5.5–7.0 7.0–9.0
P(X = x) 0.4 0.24 0.144 0.216 (cm)
Frequency
iii E(X) = (0 × 0.4) + (1 × 0.24) + (2 × 0.144) 8 25 28 31 12
(f )
+ (3 × 0.216)
Class
E(X) = 1.176 1.5 1 1 1.5 2
width (cw)
7 i Firstly, find the class widths, then each
Frequency
frequency density as shown in the table below.
density 5.33… 25 28 20.666… 6
Number of ( f/cw)
1–5 6–20 21–35 36–60 61–80
typing errors
Frequency ( f ) 24 9 21 15 42
30
Frequency density

Class width 25
5 15 15 25 20
(cw) 20

15
Frequency
4.8 0.6 1.4 0.6 2.1 10
density ( f/cw)
5

0
1 2 3 4 5 6 7 8 9
Frequency density

5
Length (cm)
4

3 10 i There must be a fixed number of trials.


2 Each trial must have exactly two discrete
1
possible outcomes, one of which is defined as
‘success’.
0 The trials must be independent with the
10 20 30 40 50 60 70 80
Number of errors probability of success the same in each trial.

ii µ ≈ (3 × 24) + (13 × 9) + (28 × 21) + (48 × 15) + (70.5 × 42) = 40.2 errors
111
1
9) + (28 × 21) + (48 × 15) + (70.5 × 42) = 40.2 errors
111
1

59

57767_P057_066.indd 59 7/5/18 4:09 PM


SUMMARY REVIEW

ii X ~ B(18, 0.15) Using the cumulative normal distribution


P(X  3) = 1 − P(X  2) tables in reverse, we can say that P(Z < 1.406)
= 0.92. Therefore:
= 1 − [P(X = 2) + P(X = 1) + P(X = 0)]
c − 14.2
 18 −1.406 =
2 16  18  17  18  18  3.6
= 1 −   × 0.15 × 0.85 +   × 0.15 × 0.8
85 +   × 0.85 
  2   1   0 −5.0616 … = c − 14.2
c = 9.14 cm (3 s.f.)
 18  18  18 
= 1 −   × 0.152 × 0.8516 +   × 0.15 × 0.8
8517 +   × 0.8518  ii Converting from x-values to z-values, we get:
  2   1   0  
16 − 14.2
z1 = = 0.5 and z 2 = 15 − 14.2 = 0.2
= 1 − 0.4796 … 3.6 3.6
= 0.520 (3 s. f.) P(15 < X < 16) = P(X < 16) − P(X < 15)
11 i P(2 paperback and 2 hardback) =
= P(Z < 0.5) − P(Z < 0.22…)
6 5 2 1 1
× × × =
8 7 6 5 28 = 0.6915 − 0.5879
But there are 4C
= 6 ways of selecting the two
2 = 0.1036
hardback books within the selection of 4. So X ~ B(7, 0.1036)
Therefore, P(X = 2) = 1 × 6 = 3 P(X  2) = 1 − P(X  1)
28 14
Alternative method: There are 8C4 = 70 different = 1 − [P(X = 0) + P(X = 1)]
6 2
possible selections. C2 × C2 = 15 of them  7  7 
involve exactly 2 hardback books. So the = 1 −   × 0.89647 +   × 0.1036 × 0.89646 
 
0  
1 
probability is 15 = 3
70 14  7  7 
= 1 −   × 0.89647 +   × 0.1036 × 0.89646 
ii P(X = 0) = P(X = 1) = 0 as they are not  0  1 
possible.
= 1 − 0.8413 …
P(X = 2) = 3 from part (i)
14 = 0.159 (3 s.f.)
6 5
P(X = 4) = × × × = 4 3 360 3
= 13 i If there are more women than men, then the
8 7 6 5 1680 14
number of women must be 4, 5 or 6.
Since probabilities sum to 1, we can say that 8C × 5C = 700
For 4 women and 2 men:
P(X = 3) = 8 .
4 2
14 For 5 women and 1 man: 8C × 5C = 280
5 1
So the probability distribution table is: For 6 women and 0 men: 8C
6 × 5C0 = 28
x 2 3 4 Therefore, the total number of ways is:
700 + 280 + 28 = 1008
P(X = x) 3 8 3
14 14 14 ii The committee could include one of the two
males (M1 or M2) or neither of them.
iii Var(X) = E(X2) − E2(X) For M1 selected and M2 unavailable, we must
=  22 × 3 + 32 × 8 + 4 2 × 3  − 32 then select 2 men from the remaining 3, and
 14 14 14 
3 women from the remaining 8. So there are
132 3C × 8C = 168 arrangements.
= −9 2 3
14
= 0.429 (3 s.f.) There are another 168 arrangements for M2
selected and M1 unavailable.
12 i
If neither of the men are selected, we must
then select 3 men from the remaining 3, and
3 women from the remaining 8. So there are
3C × 8C = 56 arrangements.
3 3
8% So the total number of ways is:
168 × 2 + 56 = 392

c 14.2
60

57767_P057_066.indd 60 7/5/18 4:09 PM


WORKED Solutions

iii If we assume that both ends of the line are 15 i There must only be two possible outcomes
occupied by women, then there are 5P2 (success and failure). Trials must be
ways of choosing those women. There are 4! independent. The probability of success must
ways of arranging the middle 4 committee be the same in each trial.
members regardless of their gender.  (banana on the 4th day) = 0.723 × 0.28
ii P
P2
5 = 0.105 (3 s.f.)
 
16 P( 25 < X < 30 ) = P  25 − 28.3 < Z < 30 − 28.3 
 4.5 4.5 
= P(– 1.556 < Z < 0.801)
= P(Z < 0.801) – (1 – P(Z < 1.556))
4! = P(z < 0.801) + P(Z < 1.556) – 1
= 0.7884 + 0.9401 – 1
So the number of ways = 5P2 × 4! = 480
= 0.729 (3 s.f.)
Alternatively, 5! × 4 = 480
17 i The probability of arriving on time is 0.2 so
14 i If the coin shows heads, then we must use X~B(96, 0.2).
the sum of the two dice, each of which has a E(X) = 96 × 0.2 = 19.2 and
minimum score of 1, so X = 1 is impossible Var(X) = 96 × 0.2 × 0.8 = 15.36
with heads. With tails, X = 1 is possible if the die So a normal approximation is X~N(19.2, 15.36)
shows a score of 1.
For the normal approximation of P(X < 20),
P(X = 1) = 1 × 1 = 1 the continuity correction means that we use
2 4 8
X = 19.5
ii To find P(X = 3), we need to consider (H, 1, 2),  
(H, 2, 1) and (T, 3). P( X < 20 ) = P  Z < 19.5 − 19.2  =
 15.36 
(1 1 1
)(
1 1 1
)(
1 1
P(X = 3) = 2 × 4 × 4 + 2 × 4 × 4 + 2 × 4 ) P(Z < 0.0765) = 0.531 (3 s.f.)
ii Let E be early, OT be on time, L be late and B be
= 1 + 1 + 1 eat a banana.
32 32 8
P(E) = 0.05
= 3
16 P(B | E) = 0.7 ⇒ P(B′ | E) = 0.3
iii To find P(X = 2), we need to consider (H, 1, 1), ∴ P(B′ ∩ E) = P(B′|E) P(E) = 0.3 × 0.05 = 0.015
and (T, 2). P(B|OT) = 0.4 ⇒ P(B′|OT) = 0.6
( 1 1 1
)(
P(X = 2) = 2 × 4 × 4 + 2 × 4
1 1
) ∴ P(B′ ∩ OT) = P(B′|OT) P(OT) = 0.6 × 0.2 = 0.12

1 1 P(B|L) = 0 ⇒ P(B′|L) = 1
= +
32 8 ∴ P(B′ ∩ L) = P(B′|L) P(L) = 1 × 0.75 = 0.75
5
= P(B′) = P(B′ ∩ E) + P(B′ ∩ OT) + P(B′ ∩ L)
32
To find P(X = 4), we need to consider (H, 1, 3), = 0.015 + 0.12 + 0.75 = 0.885
(H, 2, 2), (H, 3, 1) and (T, 4). P(OT ∩ B′) = 0.2 × 0.6 = 0.12

(
2 4 4 ) 2 4 (
P(X = 4) = 1 × 1 × 1 × 3 + 1 × 1 = 7
32 ) P (OT |B ′ ) =
P(OT ∩ B ′) 0.12
P(B ′)
=
0.885
= 0.136
Since probabilities sum to 1, P(X = 6) = 3
32
So the completed table is:
x 1 2 3 4 5 6 7 8

1 5 3 7 1 3 1 1
P(X = x)
8 32 16 32 8 32 16 32

iv P(Q ∩ R) = 0 since if the coin lands on tails,


then the 1 remaining die cannot score 7.
Therefore Q and R are mutually exclusive.

61

57767_P057_066.indd 61 7/5/18 4:10 PM


Summary REVIEW

Alternative method  n 
0.7  0  × 0.2 × 0.8 < 0.01
0 n
B  
0.3 0.8n < 0.01
0.05 B’ log 0.8n < log 0.01
E
0 nlog 0.8 < log 0.01
0.75 B
L 1 n > log0.01 (change of sign since log0.8 < 0)
log0.8
B’
0.2 n > 20.63 …
0.4
OT Therefore, the lowest possible value of n is 21.
B
0.6 20 X~Geo(0.35) ⇒ E ( X ) = 1 = 2.85… ⇒
B’ 0.35
P(OT ∩ B ′) 0.2 × 0.6
P(OT | B ′) = = Daniel should expect to ask 3 people.
P(B ′) (0.05 × 0.3) + 0.75 + (0.2 × 0.6)
P(OT ∩ B ′) 0.2 × 0.6 21 i X ~ N(μ, σ 2)
P(OT | B ′) = =
P(B ′) (0.05 × 0.3) + 0.75 + (0.2 × 0.6) 63 = 0.105 ⇒ P(X < 6) = 0.105
600
= 0.12 = 0.136 (3 sf . .) ∴ From the tables, z = – 1.253
0.885
6−µ
( )
18 i E ( x − c ) = 1957.5 = 65.25
2
30

σ = −1.253 (1)
155 = 0.2583 ⇒ P(X > 12) = 0.2583
600
E( x − c ) = 234 = 7.8
30 ∴ From the tables, z = 0.648
12 − µ

Var(x) = Var(x – c) = 65.25 – 7.82 = 4.41 ⇒ σ = 2.1 σ = 0.648 (2)

ii ∑ ( x − c ) = ∑x − ∑c (2) – (1)
6 = 1.901σ ⇒ σ = 3.156... = 3.16 (3 s.f.)
234 = ∑x − 30c
Substitute in (2)
∑x = 234 + 30c μ = 12 – 0.648 × 3.156 = 9.95 (3 s.f.)
234 + 30c = 86 ii P(Z < – 1) = 1 – 0.8413 = 0.1587
30
P(Z > 1) = 1 – 0.8413 = 0.1587
234 + 30c = 2580
P(Z < –1) or P(z > 1) is 2 × 0.1587 = 0.3174
30c = 2346
c = 78.2 0.3174 × 1000 = 317.4 ⇒ 317 feathers
Alternatively: E(X) = E(X – c) + c 22 a i 
There are 7! ways to arrange the 7 couples.
86 = 7.8 + c ⇒ c = 78.2 7! = 5040
19 i T
 here are 3 multiples of 5 in the given range and Each couple has two possible
15 possible numbers to choose from. arrangements and there are 7 couples.
P(multiple of 5) = 3 = 0.2 Therefore, the total number of
15
arrangements is 5040 × 27 = 645 210.
X~B(12, 0.2) ii There are 7! arrangements of friends, 7!
ii P(3  X  5) = P(X = 3) + P(X = 4) + P(X = 5) arrangements of partners and the whole
group can be arranged as (partners and
 12  12  12
P ( 3  X  5) =   × 0.23 × 0.89 +   × 0.24 × 0.88 +   5
× 0.87 or (friends and partners). So the
× 0.2friends)
 3  4  5
number of different arrangements is
 12  12  12 7! × 7! × 2 = 50 803 200.
3  X  5) =   × 0.23 × 0.89 +   × 0.24 × 0.88 +   × 0.25 × 0.87
 3          4   5
7! = 7 × 6 = 21
b i ways
P(3  X  5) = 0.236 … + 0.132 … + 0.053 … = 0.422 2!5! 2
ii Adults all in team ⇒ only 1 way
iii X~B(n, 0.2)
Adults all not in team ⇒ 5! = 5 ways
P(X = 0) < 0.01 4!1!
So there are 6 ways in total.
62

57767_P057_066.indd 62 7/5/18 4:10 PM


WORKED Solutions

iii At least 2 girls means either 2 girls or 3 girls. 27 i There are x yellow balls in box B (numerator).
2 girls ⇒ 3! 6! 6 × 5 The total number of balls in box B is 5 white
× = 3× = 3 × 3 × 5 = 45
2!1! 2!4! 2 balls, x yellow balls and 1 additional white ball
3 girls ⇒ 1 of the remaining 6 people
= x + 6 balls (denominator).
with the 3 girls ⇒ 6 ways
So there are 51 ways in total. ii 6
x+6 White
23 ∑ ( x − 36 ) = −60 4 White
5
∑x − ∑36 = −60 x
x+6
Yellow

∑x − 24 × 36 = −60 5
x+6 White
1
∑x = 804 5 Yellow
∑ ( x − 36 )2 = 227.76 x+1
x+6 Yellow

∑x 2 − 72∑x + ∑362 = 227.76 iii 6 = 1 ⇒ x + 6 = 18 ⇒ x = 12


x+6 3
∑x 2 2
− 72 × 804 + 24 × 36 = 227.76
iv P(Y2) = P(W1 ∩ Y2) + P(Y1 ∩ Y2)

∑x 2 = 27 011.76 = 4 × 12 + 1 × 13 = 48 × 13 = 61
5 18 5 18 90 90 90
24 i X~Geo(0.32) 1 13 13
P(Y1 ∩ Y2) 5 × 18 90 13
P(X = 7) = 0.686 × 0.32 = 0.0316 P (Y1|Y2 ) = = = =
P(Y2) 61 61 61
90 90
ii P(X < 3) = P(X = 1) + P(X = 2)
= 0.32 + 0.68 × 0.32 = 0.5376 28 i Flat screen Conventional

iii P(X > 3) = 0.683 = 0.314 (3 s.f.) 6 5 7 9


 
( )
 2 6 7 1 4 5 7
25 i Y ~N  µ, µ  P (Y < 0 ) = P  Z < 0 − µ  = P Z < − 2µ
 4  µ  µ 9 5 8 5 6
 2 
  6 4 2 1 9
 µ2 
Y ~N  µ, 
 4 
P (Y < 0 ) = P 


Z <
0 − µ

2 
( 2µ
µ  = P Z<− µ ) 7 4 10

Key: 6|7|1 means 0.76 m for flat screen


= P(Z < – 2) = 1 – P(z < 2) = 1 – 0.9722 = 0.0278
0.71 m for conventional
253
P ( X > 2.1) =
ii  = 0.03163 ii The median is 0.74 m
8000
∴ From the tables, z = 1.858 IQR = UQ − LQ =  0.77 + 0.85  −  0.67 + 0.69  = 0.81 − 0.68 = 0.13 m
 2   2 
∴ 2.1 − 2.04 = 1.858 ⇒ σ = 0. 0323 (3 s.f.)
σ
IQR = UQ − LQ =  0.77 + 0.85  −  0.67 + 0.69  = 0.81 − 0.68 = 0.13 m
26 i P(X < 73) = 0.15  2 
           2 
∴ From the tables, z = –1.036 iii Using x to represent the diagonal lengths of flat
∴ 73 − 75 = −1.036 ⇒ σ = 1.93 (3 s.f.) screen TVs: ∑x = 8.34 ⇒ µ = 8.34 = 0.927m
σ 9
ii P(X > 77) = 0.15 (by symmetry) ⇒ Y~B(8, 0.15) 7.7984
P(Y < 3) = P(Y = 0) + P(Y = 1) + P(Y = 2)
∑x 2 = 7.7984 ⇒ σ = 9 − 0.9272 = 0.0882m
 8  8  8i There must only be two possible outcomes
29
P (Y < 3) =   × 0.150 × 0.858 +   × 0.151 × 0.857 +   × 0.152 × 0.856
 0 1  2 (success and failure). Trials must be
independent. The probability of success must be
 8  8  8
P (Y < 3) =   × 0.150    
× 0.858 +   × 0.151 × 0.857 +   × 0.152 × 0.856 the same in each trial.
 0 1  2
ii P(buys on the nth call) = (1 – p)n–1p
P(Y < 3) = 0.272 … + 0.384 … + 0.237 … = 0.895

63

57767_P057_066.indd 63 7/5/18 4:10 PM


Summary REVIEW

iii When n = 2 ⇒ 0.2016 = (1 – p)p ii If the number of doctors taking their final
0.2016 = p – p2 examinations is an even number, then the
statement is always true, as the median is
p2 – p + 0.2016 = 0
positioned such that 50% of the data is below
1 ± 1 − 4(1)(0.2016) 1 ± 0.44 the median and 50% is above the median.
p= =
2 2
If the number of doctors is odd, then the
p = 0.28 or p = 0.72
number of doctors above and below the
So the maximum probability is 0.72. mean is equal, but not quite 50%, since the
30 i X~B(20, 0.05) data is discrete and the middle item of data
P(X > 1) = 1 – P(X  1) = 1 – P(X = 0) – P(X = 1) is effectively discounted. However, as the
number of doctors increases, each ‘half’ of the
 20  20
P ( X > 1) = 1 −   × 0.05 × 0.95 −   × 0.05 × 0.95 data approaches 50%. For a large number of
0 20 1 19
0 1 
doctors, the data can be treated as continuous
 20       20  and so we would say that the statement is true.
> 1) = 1 −   × 0.050 × 0.9520 −   × 0.051 × 0.9519
0 1  iii This scenario could be true in many situations.
P(X > 1) = 1 – 0.358 … – 0.377 … = 0.264 For example, suppose every doctor scored a
ii Income = 450 × 10 = $4500 different mark, it would require two or more
doctors just above 50% of the data to score
Cost = 24 × 20 = $480
the same mark and the statement is true.
If X > 1, there is a ‘profit’ of –$480 The mode is not an appropriate measure of
If X  1, there is a profit £4020 central tendency for this type of statement.
So the expected profit 3 Typically, this would be a binomial distribution
= (0.736 × 4020) + (0.264 × – 480) = $2832 with X ~ B(275, 0.9). However, the large numbers
make it impractical to use the binomial formula.
Extension Questions We know that:
np = 275 × 0.9 = 247.5 and npq = 275 × 0.9 × 0.1
1 Consider the arrangement GGBBB. If the two girls = 24.75
stay in the same position, there are 3! arrangements Since np  5 and nq  5, we can approximate the
of the boys. We also know that the following binomial distribution by a normal distribution,
arrangements of the girls will mean they are X ~ N(247.5, 24.75).
together: BGGBB, BBGGB, BBBGG, GBBBG. The last
 250 − 247.5 
arrangement works because they are standing in a P(X > 250) = P  Z > 
 24.75 
circle and the two ends meet. In each case, there will
= P(Z > 0.502 518 …)
be 3! arrangements for the boys.
Therefore, the number of arrangements for the two = 1 − P(Z < 0.502 518 …)
girls standing together is (3! × 5). = 1 − 0.6924
The total number of arrangements for the 5 = 0.3076
children is 5!
4 i T he vertical scale begins at 51 000, which gives the
Therefore, the probability of the girls standing impression that more than twice as many people
3! × 5 1
next to each other is = . voted for yes than no. To improve, raw data could
5! 4
be used or the vertical axis could start at zero.
2 i The spread of the data is the key factor if the
average referred to is the mean. This statement ii The diagram suggests that there is a
is true if the distribution is symmetrical. It is also relationship between donations to charity and
true if the average distance from the mean for mobile phone sales. The scales in each case
top 50% of the data is equal in magnitude to the are different, so there may not be a correlation.
average distance from the mean for the bottom Also, correlation does not imply causation.
50% of the data. To improve, the data is probably best shown
on two separate diagrams with a scale.

64

57767_P057_066.indd 64 7/5/18 4:10 PM


WORKED Solutions

iii The average referred to here is the median, 3C × 17C = 136 possibilities for choosing 3 of
 3 2
which is not appropriate since it does not the top 3
take into consideration sugar values that (2040 + 136)
are significantly greater than the median. = 0.140 (3 s.f.)
15504
To improve, base the calculation on the mean
10 i p + 2pq + 3pq + 4q = 2.5 ⇒ p + 5pq + 4q = 2.5 (1)
for all sweets, also considering the probability
p + pq + pq + q = 1
of each sweet being chosen.
1 p + 2pq + q = 1
5 P(1st person is selected) = n p + 2pq = 1 – q
P(2nd person is selected) = n − 1 × 1 = 1 p(1 + 2q) = 1– q
n n −1 n
1−q
P(3rd person is selected) p=
1 + 2q (2)
    = n − 1 × n − 2 × 1 = 1 Substitute (2) in (1):
n n −1 n − 2 n
1−q 1−q
+ 5q + 4q = 2.5
 his pattern continues for all n people regardless
T 1 + 2q 1 + 2q
of height. Since every person has an equal 1 – q + 5q(1 – q) + 4q(1 + 2q) = 2.5(1 + 2q)
chance of being selected, we can say that the 1 – q + 5q – 5q2 + 4q + 8q2 = 2.5 + 5q
probability of selection is independent of height.
3q2 + 3q – 1.5 = 0
6 For the first digit, there are 8 possible options. 2q2 + 2q – 1 = 0
The question tells us that 5 is not allowed and we −2 ± 4 − 4(2)(−1)
also know that to be a 5-digit number the first digit q=
4
cannot be 0.
q = −2 ± 12 = −2 ± 2 3 = −1 ± 3
For the remaining 4-digits, the only disallowed 4 4 2
number is 5, so there are 9 choices in each case. 0q1 ⇒ q = 3 −1
2
Therefore, the number of combinations is
 
1 −  32− 1 
8 × 9 × 9 × 9 × 9 = 52 488.
ii p =
 
=
( )
2− 3 −1
= −
3 3 = 3 3 − 3 = 3 −1
7 a+b = 2 ⇒ a+b=4 ⇒ b=4–a  3 − 1  2(+ 2 )
3 − 1 2 3 6 2
2 1 + 2
 2 
( a − 2)2 + (b − 2)2 = ( a − 2)2 +1(−2− a3)2− =1 a 2 − 4a + 4 + 4 − 4a + a 2 = 2a 2 − 8a + 8
σ2 = 2
p=
2  2  ( =
)
2 − 32− 1 2
= 3− 3 = 3 3 − 3 = 3 −1
( b − 2 )2 =
( a − 2) + ( 2 − a )
2 2
= a 2
− 4a + 4 + 4 − 4a + a 2
2a 2
− a +
8
= 1 + 2 2 
3 8− 1 ( )
 2 + 2 3 −      1 2 3 6 2
2 2      2 2   3 − 1  3 − 1 3 − 2 3 + 1 4 − 2 3 2 − 3
iii pq =  = = =
σ = a – 4a + 4 = (a – 2)
2 2 2  2   2  4 4 2
σ = |a – 2|         
pq =  3 − 1   3 − 1  = 3 − 2 3 + 1 = 4 − 2 3 = 2 − 3
 2  2  4 4 2
8 i 1 × 6 × 5 × 4 × 3 = 360
7 7 7 7 2401
( )
E X 2 = 3 −1 + 4
(
2− 3 ) (
2− 3 ) (
3 −1 )
ii 1 − 360 = 2041 2 2
+9
2
+ 16
2
2401 2401
9 2 or more of the top 3 students ⇒ 2 students or 3
students in the group of 5 ( )
E X 2 = 3 −1 + 4
( +9
) ( )
2 − 3       2− 3
+ 16
(
3 −1 )
2 2 2 2
For 2 students:
3 × 2 × 17 × 16 × 15 244800       ( )
E X 2 = 3 − 1 + 8 − 4 3 + 18 − 9 3 + 16 3 − 16 = 9 + 4 3
2 2
× 5C 2 =
20 × 19 × 18 × 17 × 16
For 3 students:
( )
1860480
2
E X = 3 − 1 + 8 − 4 3 + 18
2
− 9 3 + 16 3 − 16 = 9 + 4 3
2
 9+4 3
3 × 2 × 1 × 17 × 16 5
× C3 =
16320 Var ( X ) =  – 2.52 = 1.71 (3 s.f.)
20 × 19 × 18 × 17 × 16 1860 480  2 
So the required probability is:
11 i Let n be the number of red balls. Then (n + 2)
24480 1632 26112
+ = = 0.140 (3 s.f.) is the number of green balls and (2n + 2) is the
1860 480 1860 480 1860 480
total number of balls.
Alternatively:
20C possibilities = 15 504 P( different ) =  n × n + 2  +  n + 2 × n 
5  2n + 2 2n + 2   2n + 2 2n
n + 2
3C × 17C = 2040 possibilities
 for choosing 2 of
2 3
the top 3
65

57767_P057_066.indd 65 7/5/18 4:10 PM


Summary REVIEW

∴ 2n(n + 2) = 195
(2n + 2)2 392
784n(n + 2) = 195(2n + 2)2
784n2 + 1568n = 195(4n2 + 8n + 4)
784n2 + 1568n = 780n2 + 1560n + 780
4n2 + 8n – 780 = 0
n2 + 2n – 195 = 0
(n + 15)(n – 13) = 0
n = –15 or n = 13
n>0 ⇒ n = 13
Therefore, there are 13 red balls and 15 green
balls.
494 325
ii P(RGRGR) = 13 × 15 × 13 × 15 × 13 =
28 28 28 28 28 17 21 10 368
15 13 15 13 15 570 375
and P(GRGRG ) = × × × × =
28 28 28 28 28 17 21 10 368
15 13 15 13 15 570 375
P(GRGRG ) = × × × × =
28 28 28 28 28 17 21 10 368
494 325 570 375 1 064 700 38025
P ( alternating ) = + = =
17 210 368 17 210 368 17 210 368 614656
494 325 570 375 1 064 700 38025
rnating ) = + = =
17 210 368 17 210 368 614656
17 210 368            

12 P( X = x ) = n! p x (1 − p )
n− x

( )
n − x ! x !

P ( X = x − 1) = n! p x −1 (1 − p )
n −(x −1)

( )
n − ( x − 1) !(x − 1)!

(n − x )! x ! ( )
n! px 1 − p
n− x
P( X = x )
=
P ( X = x − 1) n! p x −1 (1 − p )
n −(x −1)

(n − ( x − 1))!(x − 1)!
n ! ( n − x + 1)! ( x − 1)! p x (1 − p )
n− x
P( X = x )
=
P ( X = x − 1) n ! ( n − x )! x ! p x −1 (1 − p )
n − x +1

P( X = x ) p(n − x + 1)
=
P ( X = x − 1) x(1 − p)

66

57767_P057_066.indd 66 7/5/18 4:10 PM

You might also like